Top Posters
Since Sunday
k
1
1
1
o
1
A free membership is required to access uploaded content. Login or Register.

Ch08 Pathophysiology.docx

Uploaded: 6 years ago
Contributor: BritishGent
Category: Nursing
Type: Other
Rating: N/A
Helpful
Unhelpful
Filename:   Ch08 Pathophysiology.docx (143.32 kB)
Page Count: 76
Credit Cost: 1
Views: 102
Last Download: N/A
Transcript
Chapter 8 Pathophysiology Unit Summary After students complete this chapter, they will have an understanding of cellular changes in response to stressors. The understanding of what happens when the cellular system can no longer maintain homeostasis is a key component of patient evaluation and treatment. National EMS Education Standard Competencies Pathophysiology Integrates comprehensive knowledge of pathophysiology of major human systems. Knowledge Objectives 1. Define pathophysiology, and discuss its scope. (p 335) 2. Explain the function of each of the three main components of a human cell: the cell membrane, the cytoplasm, and the nucleus. (pp 335-336) 3. Discuss how the body maintains homeostasis. (p 337) 4. Describe the characteristics of the four basic tissue types—epithelial, connective, muscle, and nerve tissue—and specify where each is found in the body. (pp 336-337) 5. Explain the function of common ligands, including medications, hormones, neurotransmitters, and electrolytes. (p 339) 6. Compare atrophy, hypertrophy, hyperplasia, dysplasia, and metaplasia as means of cellular adaptation. (pp 339-340) 7. Analyze the functions of water in the body. (pp 340-341) 8. Explain the concepts of osmotic pressure and membrane permeability. (p 341) 9. Explore the causes, clinical manifestations, assessment, and management of edema. (p 342) 10. Survey the mechanisms by which fluid and electrolyte balance are maintained in the body. (p 342) 11. Explain the physiologic consequences of imbalances in sodium, potassium, calcium, phosphate, and magnesium. (pp 342-346) 12. Become familiar with the concepts of acid, base, and pH. (p 347) 13. Explain how proteins, phosphate ions, and bicarbonate (HCO3? ) buffer pH imbalances in the body. (pp 347-349) 14. Compare the four main clinical presentations of acid-base disorders: respiratory acidosis, respiratory alkalosis, metabolic acidosis, and metabolic alkalosis, and describe the clinical presentation that might be associated with a mixed acid-base disorder. (pp 349-352) 15. Outline how cellular injury occurs in patients with hypoxia, chemical exposures, infection (sepsis), immunologic exposures ( hypersensitivity reactions), inflammatory conditions, genetic disorders, nutritional imbalances, physical damage (mechanical injury), and other harmful exposures, such as extremes of hot and cold. (pp 352-356) 16. Examine the concept of apoptosis. (p 356) 17. Analyze the controllable and uncontrollable risk factors that intersect in order to cause disease. (p 357) 18. Outline how incidence, prevalence, morbidity, and mortality data are used in analyzing disease risk. (p 357) 19. Become familiar with autosomal dominant and autosomal recessive patterns of inheritance. (pp 358, 359) 20. Analyze risk factors for cancer and cardiovascular disease. (pp 359-360, 362-363) 21. Describe how the body synthesizes hemoglobin. (p 361) 22. Identify several common renal, gastrointestinal, and neuromuscular disorders. (pp 363-365) 23. Define perfusion, and explain the physiologic consequences of hypoperfusion. (p 366) 24. Analyze the mechanisms by which the body compensates for hypoperfusion. (p 366) 25. Discuss the causes of central and peripheral shock, including cardiogenic, obstructive, hypovolemic, and distributive shock. (pp 366-369) 26. Outline the management of a patient in shock. (p 369) 27. Describe multiple organ dysfunction syndrome. (pp 369-370) 28. Examine the body’s three lines of defense against pathogens: anatomic barriers, the immune response, and the inflammatory response. (pp 370-383) 29. List the functions of the five general types of white blood cells: basophils, eosinophils, monocytes, neutrophils, and lymphocytes. (pp 372, 373) 30. Describe the function of macrophages. (p 372) 31. Describe the function of mast cells. (p 373) 32. Compare humoral immunity with cell-mediated immunity. (pp 374-378) 33. Explain how plasma protein systems—the complement system, the coagulation (clotting) system, and the kinin system—modulate the inflammatory response. (p 380) 34. Compare wound healing by primary intention with wound healing by secondary intention. (pp 381, 383) 35. Explain why hypersensitivity reactions sometimes occur, and outline the four types of hypersensitivity reactions. (pp 383, 384) 36. List several autoimmune reactions, and explain blood group incompatibility. (pp 385, 386) 37. Compare inherited and acquired immunodeficiencies. (pp 386, 387) 38. List the stages of the general adaptation syndrome, and explore the relationship between stress and disease. (pp 387-389) Skills Objectives There are no skills objectives in this chapter. Readings and Preparation Review all instructional materials including Chapter 8 of Nancy Caroline’s Emergency Care in the Streets, Seventh Edition, and all related presentation support materials. Support Materials • Lecture PowerPoint presentation • Case Study PowerPoint presentation • Human body diagram and cellular structure diagram Have several copies for each student, as they can serve as a template for many activities and assessments. Enhancements • Direct students to visit the companion website to Nancy Caroline’s Emergency Care in the Streets, Seventh Edition, at http://www.paramedic.emszone.com for online activities. • Web links Learn more about muscle physiology and design. http://muscle.ucsd.edu/musintro/jump.shtml This link contains many video lessons that will help students understand the different concepts relating to biology. http://www.brightstorm.com/science/biology/ This link contains easy-to-understand descriptions and illustrations of cell structures and their functions. http://library.thinkquest.org/12413/structures.html • Content connections: The chapter titled Anatomy has a direct correlation to this chapter. The assessment of every patient depends on the student’s thorough understanding of anatomy and physiology. • Cultural considerations: Some diseases occur more often in men, and others occur more often in women. Diseases such as lung cancer, gout, and Parkinson disease are more prevalent in men. Women are more likely to have diseases such as osteoporosis, rheumatoid arthritis, and breast cancer. Some diseases present differently in women compared with men. For example, the hormones found in a premenopausal woman have been shown to have a protective effect in major head trauma and certain cardiac conditions. Studies are under way to investigate this finding, which could lead to the possibility of administering female sex hormones to male cardiac arrest patients to improve survival. Finally, genetic disorders are related to a person’s sex when a defective gene is located on a sex chromosome. Most sex-linked disorders are X linked. X-linked disorders may be either recessive or dominant. Teaching Tips • Images are integral to the retention of material. Prepare ahead of time, and consider incorporating one or more of the web links mentioned in the “Enhancements” section. • Choosing a variety of approaches, including charts, interactive exercises, and group- and self-assessments, will allow students to organize the new information and identify areas needing more review. Including multiple activities with visual components will reduce learning time and increase retention. See the “Student presentation” and “Group activities” sections for suggestions. This chapter is particularly visual friendly, and with instructor guidance, students can engage in creative, fun learning. Unit Activities Writing activities: Using the systems researched in the “Group activities” section, or selecting another system, structure, or organ, have each student or group of students write one or two paragraphs on one illness and one injury that could directly affect the particular organ/structure. What would they expect to see? Open up the presentations to class discussion if time allows. Student presentations: Ask students to present their writing assignment to the class, requiring them to also prepare a one-page self-assessment to be distributed with the presentation. Group activities: Organize the class into groups. Assign each group to explain one of the following imbalances: sodium, potassium,calcium, phosphate, or magnesium. Visual thinking: Assign a cellular structure to each group with instructions to create a presentation for the whole class on that system. Each group should be given the same guidelines and questions that must be answered within the presentation. Questions to consider include: What is the location of the structure? How does each structure work independently? How does each structure interact with the other parts of this system? What other systems interact with this structure? Pre-Lecture You are the Medic “You are the Medic” is a progressive case study that encourages critical-thinking skills. Instructor Directions Direct students to read the “You Are the Medic” scenario found throughout Chapter 8. • You may wish to assign students to a partner or a group. Direct them to review the discussion questions at the end of the scenario and prepare a response to each question. Facilitate a class dialogue centered on the discussion questions and the Patient Care Report. • You may also use this as an individual activity and ask students to turn in their comments on a separate piece of paper. Lecture I. Introduction A. The human body is made up of cells, tissues, and organs. 1. Biology is the study of living organisms with regard to their origin, growth, structure, behavior, and reproduction. B. Pathophysiology 1. Study of the functioning of an organism in the presence of disease 2. Derived from Greek words a. Pathos: Suffering b. Physis: Form 3. When the condition or functioning of a cellular system breaks down in response to stressors and homeostasis can no longer be maintained, disease can occur. a. Understanding the cause of the disease process can help in evaluation and treatment. 4. To understand how disease may alter cellular function, you must understand normal cellular structure and function. II. Review of the Basic Cellular Systems A. Cells 1. Basic self-sustaining unit of the human body 2. Become specialized through differentiation as they grow and mature 3. May perform one function or work with other cells to act similarly to multicellular organisms 4. Groups of cells form tissues. a. Tissues make up organs. b. Groups of organs make up organ systems. 5. Excluding mature red blood cells and platelets, nearly all cells of higher organisms have three main components: a. Cell membrane b. Cytoplasm i. Contains the cell’s internal components (organelles) c. Nucleus 6. Cell membrane a. Made up of fat and protein b. Surrounds the cell c. Protects the nucleus and organelles 7. Organelles a. Functional structures within the cell’s cytoplasm (fluid) b. Operate in a cooperative and organized manner to maintain life of the cell 8. Organelles contain the following components: a. Ribosomes i. Contain ribonucleic acid (RNA) and protein (a) RNA controls cellular activities. ii. Interact with RNA from other parts of the cell to join amino acid chains together to form proteins iii. Create rough endoplasmic reticulum when they attach to an endoplasmic reticulum b. Endoplasmic reticulum i. Network of tubules, vesicles, and sacs ii. Rough endoplasmic reticulum: involved in building proteins iii. Smooth endoplasmic reticulum: involved in building lipids (fat) (a) For example, those found in cell membranes c. Golgi complex i. Located near the nucleus ii. Involved in synthesis and packaging of carbohydrates and complex protein molecules d. Lysosomes i. Membrane-bound vesicles containing digestive enzymes ii. Function as intracellular digestive system (a) Break down debris and bacteria that has to be taken into the cell e. Peroxisomes i. Found in high concentrations in the liver ii. Neutralize toxins like alcohol f. Mitochondria i. Small organelles ii. May be rod like or spherical in shape iii. Metabolic center for the cell iv. Produce adenosine triphosphate (ATP) (a) Major energy source for the cell g. Nucleus i. Contains two types of genetic material (a) Deoxyribonucleic acid (DNA) (1) Contained in the chromosomes, which appear as a network of granules called nuclear chromatin (b) Ribonucleic acid (RNA) (1) Contained is spherical structures called nucleoli ii. Surrounded by a membrane called the nuclear envelope iii. Embedded in the cytoplasm B. Tissues 1. Groups of similar cells working together to perform a common function 2. There are four basic types. a. Epithelial tissue (epithelium) i. Covers external surfaces of the body ii. Lines hollow organs within the body iii. Provides a protective barrier iv. Necessary for: (a) Absorption of nutrients in the intestines (b) Secretion of various body substances, such as sweat v. Endothelial cells: epithelial cells that line the blood vessels (a) Helps regulates blood flow (b) Have a role in coagulation b. Connective tissue i. Binds other types of tissue to one another ii. Separated by a nonliving extracellular matrix consisting of protein fibers, nonfibrous proteins, and fluid iii. Collagen is the main protein in the extracellular matrix. (a) There are at least 12 types of collagen. (1) Types I, II, III are most abundant. (b) Alteration of collagen structure caused by abnormal genes or processing of collagen proteins occurs in diseases like scurvy iv. Bone and cartilage are subtypes. v. Adipose tissue (a) Special type of connective tissue (b) Contains large amounts of lipids vi. Blood is also consider a connective tissue. c. Muscle tissue i. Characterized by its ability to contract ii. Enclosed by fascia (a) Envelope of fibrous connective tissue that encapsulates individual muscles iii. Muscles overlie the framework of the skeleton. iv. Classified by structure (a) Striated muscle: Muscle tissue in which microscopic bands (striations) can be seen (b) Smooth muscle: Muscle without striations v. Classified by function (a) Voluntary: Consciously controlled (b) Involuntary: Not normally under conscious control vi. Structural and functional classifications are combined in the categorization of three muscle types. (a) Skeletal muscle (striated voluntary) (1) Most of the voluntary muscles used in everyday activity (b) Cardiac muscle (striated involuntary) (1) The heart consists of cardiac muscle. (2) Contractile ability (3) Ability to generate electrical impulses (c) Smooth muscle (nonstriated involuntary) (1) Lines most glands, digestive organs, lower airways, vessels (2) When the brain signals vasoconstriction in response to an environmental stimulus the vessels in the periphery react. (3) Example: Smooth muscle of the bronchioles may constrict during an asthma attack. d. Nerve tissue i. Transmits nerve impulses ii. Central nervous system (CNS) consists of brain and spinal cord (a) Peripheral nerves extend from the brain and spinal cord, exiting from between the vertebrae to various parts of the body. iii. Neurons (a) Main conducting cells of nerve tissue (b) Cell body of the neuron is the site of most cellular functions iv. Dendrites receive electrical impulses from the axons of other nerve cells. (a) Conduct them toward the cell body v. Axons typically conduct electrical impulses away from the cell body vi. Each neuron has only one axon, but it may have several dendrites. vii. Synapse: Gap that separates nerve cells viii. Electrical impulses travel down the nerve and trigger the release of neurotransmitters. (a) Neurotransmitters carry the impulse from axon to dendrite. C. Homeostasis 1. Adaptive responses to various stimuli that allow the cells and tissues to respond and function in stressful environments in an effort to maintain equilibrium 2. From the Greek words “same” and “steady” 3. Also called the dynamic steady state 4. Physiologic cell turnover: The process in which older cells are eliminated and replaced by newer ones a. Apoptosis: Normal, genetically programmed cell death 5. Maintained because normal regulatory systems are counterbalanced by counterregulatory systems a. For every cell, tissue, or organ that performs one function; there is always one component that performs the opposing function i. Example: Sympathetic and parasympathetic components of the autonomic nervous system ii. Other examples: Control of body temperature, regulation of pH and acid-base balance, balance of hydration in the cells and body of the organism 6. Regulatory systems communicate within the body mainly at the cellular level. a. Cells communicate electrochemically through cell signaling. i. Release of molecules that bind to protein receptors on cell surfaces (a) Triggers a chemical reaction in the cells that initiate a biological reaction (b) When the action has been completed, the opposing system is alerted to discontinue the action. (1) This is called feedback inhibition or negative feedback. 7. Receptors are specialized depending on the role they perform. a. Adrenergic receptors i. Associated with the sympathetic nervous system ii. Stimulated by epinephrine and norepinephrine iii. Activation causes a sympathetic response, such as vasoconstriction or vasodilatation iv. Alpha and beta receptors are in this class. b. Baroreceptors and chemoreceptors are involved in the regulation of heart function. i. Baroreceptors respond to changes in pressure, usually within the heart or main arteries. ii. Chemoreceptors sense changes in the chemical composition of the blood, especially reduced oxygen and carbon dioxide levels. iii. When abnormalities are sensed, they transmit nerve signals to appropriate organs releasing hormones to correct the situation. 8. Homeostasis and the house a. Heat in a house is maintained by a furnace, which compensates for heat loss. b. A thermostat monitors the internal temperatures and switches the furnace on and off in response to temperature changes. c. Similarly the body constantly generates heat through cellular process. i. Five primary mechanisms help the body reduce excess temperatures (a) Convection (b) Conduction (c) Radiation (d) Evaporation (e) Respiration d. The body’s thermostat balances the generation of heat with the elimination of heat. 9. Human body maintains homeostasis by balancing what it takes in with what it puts out a. Example: The body takes in food and water; utilizes the nutrients, proteins, sugars, and oxygen; then eliminates the unnecessary chemicals and by-products through respiration, sweating, urination, and defecation. 10. When cell signaling is interrupted, disease occurs. a. Counterregulatory mechanisms are rendered ineffective. b. Regulatory systems begin to operate autonomously. 11. Excessive output can rapidly upset homeostasis. a. Examples: Diarrhea, profuse perspiration, changes in water intake b. Degree of fluid loss to upset homeostasis and cause illness depends on: i. Patient size, age, and underlying medical conditions (a) Healthy adult requires loss of 30% of total body fluid (b) Small child requires only 10% to 15% c. Fluid therapy is one of the basics in resuscitation. D. Ligands 1. Molecules that bind to receptors in the body to form more complex structures a. Can be produced by the body: Endogenous ligands b. Can be administered as a drug: Exogenous ligands 2. Common ligands include: a Hormones i. Substances formed in specialized organs or groups of cells and carried to another organ or group of cells in the same organism ii. Endocrine hormones (thyroid hormones and adrenal steroids) are carried to their target organs or cell group by the blood. iii. Exocrine hormones (stomach acids and perspiration) reach their target via a specific duct that opens into an organ. iv. Paracrine hormones (histamine released during allergic and inflammatory reactions) diffuse through intercellular spaces to reach their target organ. v. Autocrine hormones are hormones that act on the cell from which it has been secreted. b. Neurotransmitters i. Proteins that affect signals between cells of the nervous system c. Electrolytes i. Dissolved mineral salts that dissociate in solution, yielding ions ii. Examples: Sodium, potassium, calcium, and chloride iii. Play an important role in cell signaling and in generating the nervous system action potential iv. Ions can be: (a) Cations: Positively charged (b) Anions: Negatively charged v. Number of positive ions is equal to negative ions (a) In disease the concentrations of individual ions may vary, but electrical neutrality is always maintained. III. Adaptations in Cells and Tissues A. When exposed to adverse conditions, cells undergo a process to attempt to protect themselves from injury. 1. Change can be permanent or temporary 2. Atrophy a. Decrease in cell size due to a loss of subcellular components i. Leads to a decrease in the size of the tissue and organ b. The actual number of cells remains unchanged. c. Decreased size represents an attempt to cope with a new steady state with less-than-favorable conditions. 3. Hypertrophy a. Increase in the size of the cells due to synthesis of more subcellular components i. Leads to an increase in tissue and organ size 4. Hyperplasia a. Increase in the actual number of cells in an organ or tissue i. Usually results in an increase in size of the organ or tissue 5. Dysplasia a. Alteration in the size, shape, and organization of cells b. Most often found in epithelial cells that have undergone irregular, atypical changes 6. Metaplasia a. Reversible, cellular adaptation in which one adult cell type is replaced by another IV. The Cellular Environment A. Distribution of body fluids 1. Cellular environment changes with aging, exercise, pregnancy, medications, disease, and injury 2. Body fluids contain water, sodium, chloride, potassium, calcium, phosphorus, and magnesium, plus other substances. 3. Total body weight is approximately 50% to 70% fluids. a. Average male is 60% fluids. b. Average female is 50% fluids. c. Intracellular fluid: 45% of body weight d. Extracellular fluid: 15% of body weight i. Interstitial fluid: Extracellular fluid surrounding tissue cells, including cerebrospinal fluid and synovial fluid ii. Intravascular fluid: Extracellular fluid found within the blood vessels but outside the cells B. Fluid and water balance 1. The average adult takes in about 2,500 mL of water per day. a. 60% from drinking b. Another 30% from water in foods c. Remaining 10% is a by-product of cellular metabolism 2. Most water is lost in the form of urine (60%). a. 28% is lost through the skin and lungs. b. 6% is lost in feces. c. 6% is lost through sweat. 3. Water and dissolved particles move between cells as well as between blood vessels and connective tissues. a. Two general methods: Passive transport and active transport 4. Osmosis a. Movement of water or another solvent from a region of high water concentration to one of lower water concentration b. Pressure occurs when two solutions of different concentrations are separated by a semipermeable membrane. i. Moves from region of low pressure to region of higher pressure c. Hypertonic solution: The solution with a higher solute concentration d. Hypotonic solution: The solution with a lower solute concentration e. Isotonic solution: Solutions with equal solute concentrations 5. Intracellular fluid volume controlled in two ways: a. By the proteins and organic compounds that cannot escape through the cell membrane b. By the sodium-potassium membrane pump i. Responsible for keeping cell rupture in check ii. Maintains the cell’s electrical potential (a) Removes three sodium ions for every two potassium ions that are moved back into the cell. iii. Failure causes sodium to accumulate and the cell to swell 6. Plasma a. Makes up about 55% of the blood b. Composed of 91% water and 9% plasma proteins (albumin, globulin, fibrinogen, and prothrombin) c. Starling hypothesis explains the movement of water between plasma and interstitial fluid i. Normal conditions: The amount of fluid filtering outward through the arterial ends of the capillaries is equal to the amount of fluid that is returned to the circulation by reabsorption at the venous ends of the capillaries. d. Equilibrium between capillary and interstitial space is controlled by four forces. i. Capillary hydrostatic pressure pushes water out of the capillary into the interstitial space. ii. Capillary colloidal osmotic pressure is generated by dissolved proteins in the plasma that are too large to penetrate the capillary membrane. iii. Tissue hydrostatic pressure opposes the passage of fluids from the capillary into the interstitial space. iv. Tissue colloidal osmotic pressure pulls fluid into the interstitial space. e. Capillary and membrane permeability serve an important role in the movement of fluid and in the development of edema. i. Increased permeability: Capillaries and membranes are more likely to leak. ii. Decreased permeability: Capillaries and membranes are less likely to leak. 7. Edema a. Occurs when excess fluid builds up in the interstitial space i. Peripheral edema (ankles and feet) is the most common form. ii. Severe edema may be caused by long-standing lymphatic obstruction. iii. Sacral edema may occur in bedridden patients. iv. Ascites: Abnormal accumulation of fluid in the peritoneal cavity b. May have several causes: i. Increased capillary pressure from: (a) Arteriolar dilation (b) Venous obstruction (c) Increased vascular volume (d) Premenstrual sodium retention (e) Pregnancy (f) Environmental stress (g) Effects of gravity from prolonged standing ii. Decreased colloidal osmotic pressure in the capillaries from: (a) Decreased production of plasma proteins (b) Increased loss of plasma proteins iii. Lymphatic vessel obstruction due to infection or diseases of the lymphatic structures or removal of lymph nodes c. Clinical manifestations i. May be local or generalized ii. Pulmonary edema: For cardiac reasons or following near-drowning incident or overdose (a) Excess fluid in the lungs impairs the diffusion of oxygen into the capillaries causing hypoxia. d. Assessment and management i. Must perform an in-depth assessment including: (a) Auscultation of lung sounds (b) Evaluation for pedal or sacral edema and jugular venous distension (c) ECG and vital signs ii. Determine a patient’s medical history and current and past medications. iii. Treatment may include: (a) Diuretics (b) Nitrates (c) Continuous positive airway pressure (CPAP) (d) High-flow oxygen (e) Advanced airway placement C. Fluid and electrolyte balance 1. Maintained through a variety of factors a. The thirst mechanism and release of antidiuretic hormone (ADH), also known as vasopressin, are the most important. b. The renin-angiotensin-aldosterone system (RAAS) plays an important role in water homeostasis. 2. Hydration is monitored by three types of receptors: a. Osmoreceptors monitor extracellular fluid osmolarity. i. Sensors are located primarily in the hypothalamus. ii. Stimulate the release of ADH when extracellular fluid osmolarity is too high b. Volume-sensitive receptors are located in the atria. i. When intravascular fluid volume increases, the atria are stretched causing the release of atrial natriuretic proteins. c. Baroreceptors are found primarily in the carotid artery, aorta, and kidneys. i. Sensitive to changes in blood pressure 3. The most potent stimulation for the release of ADH is increase in blood osmolarity. a. ADH stimulates the kidneys to reabsorb water, decreasing osmolarity. 4. Regulation of sodium, chloride, and water balance a. Sodium: Most common cation (positively charged ion) in the body i. The average adult has 60 mEq of sodium for each kilogram of body weight; most is found in the extracellular fluid. ii. Intracellular sodium: Transported out of the cell by the sodium-potassium pump iii. Sodium plays an important role in the regulation of the body’s acid-base balance. iv. 500 mg per day meets the body’s needs. (a) In the United States, the average adult ingests approximately 3,400 mg per day. (b) AHA recommends to cut it to 1,500 mg/d v. Primary regulator of sodium is the RAAS. (a) Excess sodium is excreted into the urine. (b) When sodium is low, kidneys resorb sodium. vi. Renin: Protein that is released by the kidneys into the bloodstream in response to changes in: (a) Blood pressure (b) Blood flow (c) Amount of sodium in tubular fluid (d) Glomerular filtration rate vii. Renin converts angiotensinogen to angiotensin I. (a) Angiotensin I is converted in the lungs to angiotensin II. (1) Angiotensin II stimulates sodium resorption by the renal tubules and constricts the renal blood vessels, slowing kidney blood flow and decreasing the glomerular filtration rate. (2) Angiotensin II is also responsible for stimulating the secretion of the adrenal hormone aldosterone. (3) Aldosterone acts on the kidneys to increase the reabsorption of sodium into the blood and enhance the elimination of potassium in the urine. viii. An increase in production of natriuretic proteins occurs when the body contains too much sodium and water. (a) Inhibit ADH and promote excretion of sodium and water by the kidneys ix. Chloride: Important anion (negatively charged ion) (a) Assists in regulating the acid-base balance and is involved in the osmotic pressure of the extracellular fluid (b) Chloride will follow sodium in many cases. b. Change in water content can cause a cell to shrink or swell. i. Tonicity: Tension exerted on a cell due to water movement across the cell membrane ii. In isotonic solution, cells neither shrink nor swell. iii. In hypertonic solution, water is pulled out of the cells and they shrink. iv. In hypotonic solution, cells swell. v. Isotonic fluid deficit: Decrease in extracellular fluid with proportionate losses of sodium and water vi. Isotonic fluid excess: Proportionate increase in both sodium and water in the extracellular fluid compartment (a) Causes include kidney, heart, and liver failure vii. When dehydration exists, orthostatic hypotension and decreased urine output (oliguria) are common. viii. When sodium level is very high, hyperthermia, delirium, and coma may be seen. c. Electrolyte imbalances i. Sodium (a) Normal serum sodium levels are 136 to 144 mEq/L. (b) Hypertonic fluid deficit: Caused by excess body water loss without a proportionate sodium loss (1) Results in hypernatremia (serum sodium level greater than 145 mEq/L) (c) Hypotonic fluid deficit: Caused by excessive sodium loss with less water loss (1) Results in hyponatremia (serum sodium level less than 135 mEq/L) (d) Causes of hyponatremia and hyponatremia include excess sweating from hot environmental conditions, vomiting, diarrhea, and inappropriate use of diuretics. (e) Clinical findings typically depend not only on absolute sodium levels but also on the period during which the abnormality developed. ii. Potassium (a) Necessary for: (1) Neuromuscular control (2) Regulation of the three types of muscles (3) Acid-base balance (4) Intracellular enzyme reactions (5) Maintenance of intracellular osmolarity (b) Normal serum potassium levels are 3.5 to 5 mEq/L. (c) Hypokalemia: Decreased serum potassium level (1) Common causes include decreased potassium intake, potassium shifts into cells, renal potassium losses, and extra renal potassium losses. (2) Frequent complaints include muscular weakness, fatigue, and muscle cramps. (3) Flaccid paralysis, hyporeflexia, and tetany may occur. (4) ECG shows decreased amplitude and broadening of T waves, prominent U waves, premature ventricular contractions and other dysrhythmias, and depressed ST segments. (d) Hyperkalemia: Elevated serum potassium level (1) Common causes include spurious causes (example repeated fist-clenching during phlebotomy), decreased excretion (example renal failure,) shifts of potassium from within the cell (such as burns), and excessive intake of potassium. (2) May lead to muscle weakness and flaccid paralysis (3) ECG changes include peaked T waves, widening of the QRS complex, and dysrhythmias. (4) Can be life threatening (5) Calcium administered intravenously immediately antagonizes cardiac conduction abnormalities. (6) Bicarbonate, insulin, and albuterol work in a 15- to 30-minute period. iii. Calcium (a) Majority is found in bone and teeth (b) Provides strength and stability for the collagen and ground substance forming the matrix of the skeletal system (c) Absorbed from the intestine in a process that depends on the presence of vitamin D (d) Normal serum calcium level is 8.5 to 10.5 mg/dL (e) Hypocalcemia: Decreased serum calcium level (1) Causes include decreased intake or absorption, increased loss (such as alcoholism), endocrine disease, and sepsis. (2) Symptoms may include spasm of skeletal muscle causing cramps and tetany, laryngospasm, and seizures and abnormal sensations (paresthesias) of the lips and extremities. (f) Hypercalcemia: Increased serum calcium level (1) Causes include increased intake or absorption (such as excess antacid ingestion), endocrine disorders, neoplasms, and miscellaneous causes. (2) Symptoms include constipation and frequent urination (polyuria). (3) Stupor, coma, and renal failure may develop in severe cases. iv. Phosphate (a) Intracellular anion essential to many functions (b) Hypophosphatemia: Decrease in serum phosphate levels (1) Caused by decreased supply or absorption, excessive loss of phosphate (such as use of diuretics), intracellular shift of phosphorus (such as after the administration of glucose), electrolyte abnormalities, and abnormal losses followed by inadequate repletion (such as diabetic ketoacidosis). (2) Symptoms include muscle weakness, decreased deep tension reflexes, mental obtundation, and confusion. (3) Treatment involves oral replenishment or IV phosphate replacement. (c) Hyperphosphatemia: Increase in serum phosphate level (1) Causes include massive loading of phosphate into the extracellular fluid (such as excess vitamin D) and decreased excretion into the urine (such as renal failure). (2) Symptoms may include tremor, paresthesia, hyporeflexia, confusion, seizures, muscle weakness, stupor, coma, hypotension, heart failure, and prolonged QT interval. v. Magnesium (a) Second most abundant intracellular cation (b) Normal range of serum magnesium is 1.5 to 2 mEq/L. (c) Hypomagnesemia: Decreased serum magnesium level (1) Causes include diminished absorption or intake, increased renal loss, and miscellaneous causes (such as diabetes or respiratory alkalosis). (2) Common symptoms include weakness, muscle cramps, and tremor. (3) Treatment consists of IV fluids containing magnesium. (d) Hypermagnesemia: Increased serum magnesium level (1) Results from kidney insufficiency and the inability to excrete the amount of magnesium taken in (2) Symptoms include muscle weakness, decreased deep tendon reflexes, mental obtundation, and confusion. V. Acid-Base Balance A. Overview 1. Acid is any molecule that can give up a hydrogen ion. 2. Base, or alkali, is any molecule that can accept a hydrogen ion. 3. Acidity or basicity (alkalinity) of a solution is determined by the amount of free hydrogen in the solution. 4. The pH of a solution is a measurement of the level of its acidity or alkalinity. a. Acidic solution is when pH is less than 7 b Alkaline solution is when pH is greater than 7 c. Normal pH is between 7.35 and 7.45. 5. Hypoventilation, hyperventilation, and hypoxia can disrupt the acid-base balance. 6. Death occurs when the pH drops below 6.9 or rises above 7.8. 7. Changes in pH are exponential not linear B. Disturbances of acid-base balance 1. Acid and bases neutralize each other and must remain in balance. a Acidosis is an increase in extracellular H+ ions. b. Alkalosis is a decrease in extracellular H+ ions. 2. Disturbances of acid-base balance are associated with disturbances in potassium balance. a. Kidney transport system moves H+ and K in opposite directions. i. Acidosis: Excretes H+ and resorbs K ii. Alkalosis: Excretes K and resorbs H+. 3. Calcium ions also shift in response to the influx of hydrogen. a. A high serum calcium level decreases the rate of neural transmission. b. A low serum calcium level leads to hypersensitive neurons and an accelerated rate of neural transmission. C. Buffer systems 1. Buffers: Molecules or compounds that modulate changes in pH by neutralizing excessive acids or bases a. When buffers are absent, the addition of acid will cause an abrupt change in pH. 2. pH scale ranges from 0 to 14. a. Acid or base can be classified as weak or strong depending on how completely it dissociates in water. i. Any solution approaching a pH of 14 is considered a strong base. ii. Any solution approaching a pH of 0 is considered a strong acid. 3. Buffer systems include proteins, phosphate ions, and bicarbonate. a. Most physiological buffers combine with H+. b. Protein buffering refers to charged proteins in cells accepting or donating hydrogen ions. i. Regulates acid-base balance c. Body responds to normal shifts in pH by absorbing or releasing the necessary amount of acid into the blood. i. Problems occur when the amount of acid is too great for the buffer system. d. Three primary buffer systems: Circulating bicarbonate buffer component, the respiratory system, and the renal system i. Body’s fastest means of restoring acid-base balance is the bicarbonate content of the intracellular and extracellular fluid ii. Respiratory system: When excessive acid builds up it is eliminated through the lungs when carbon dioxide is expelled. iii. Renal system: Regulates pH by filtering out hydrogen and retaining bicarbonate when necessary or by doing the reverse e. Equation to illustrate the balance among the components: i. CO2 + H2O (respiratory component) H2CO3 (circulating bicarbonate component) HCO3– (renal component) 4. Circulating bicarbonate buffer component a. Excess acid combines with bicarbonate, forming carbonic acid (H2CO3). i. Rapidly dissociates into water and carbon dioxide (CO2), which is exhaled ii. Because the acid is eliminated as water and carbon dioxide, the total pH does not change significantly. b. Carbonic acid is a weak acid, giving up the extra H+ ion to reform as bicarbonate ion. i. H2CO3 H+ HCO3– ii. The extra H+ ion is easily converted during metabolism into compounds that are easily expelled from the body. 5. Respiratory buffer component a. The body can eliminate excess H+ ions to create water and carbon dioxide, which can be expelled as gases from the lungs. i. H2CO3 CO2 + H2O b. Function of the respiratory system: To distribute oxygen to the body for aerobic metabolism and remove excess carbon dioxide from the blood i. Carbonic acid is formed when carbon dioxide combines with circulating water. ii. Chemoreceptors sense the rising levels of carbonic acid and signal the respiratory center to increase the respiratory rate. c. Slower to respond to changes than the circulating bicarbonate buffer component d. Any inhibition of respiratory function can lead to acid retention and acidosis. D. Alkalosis can develop if the respiratory rate is too high or the volume too large. 1. Renal buffer component a. Kidneys maintain hemostasis by retaining certain products and filtering out others. b. Could take hours or even days for the system to restore pH to normal c. Controls increasing acid levels by excretion i. Kidneys excrete acid in an ionic form in the urine. (a) H2CO3 H+ + HCO3– (b) If urine output drops, acidosis can develop. (c) If urine output becomes excessive, alkalosis can develop. E. Types of acid-base disorders 1. Fluctuations in pH due to bicarbonate levels result in metabolic acidosis or alkalosis. 2. Fluctuations in pH due to respiratory disorders result in respiratory acidosis or alkalosis. 3. When an acid-base disorder is not immediately correctable by the body’s buffering systems, the body initiates compensatory mechanisms. a. Patient management often involves treating more than one form of acid-base imbalance. 4. There are four main clinical presentations: a. Respiratory acidosis i. Always related to hypoventilation ii. Compensatory mechanism is the renal buffer system iii. Causes include airway obstruction, cardiac arrest, OD of CNS depressant, near drowning, respiratory arrest, pulmonary edema, closed head injury, chest trauma, and carbon monoxide poisoning. iv. Hypoventilation can quickly develop a potentially fatal acidosis making it impossible for the slow reacting renal system to compensate. v. Signs and symptoms include systemic or cerebral vasodilatation; headaches; red, flushed skin; CNS depression; bradypnea; nausea and vomiting; and hypercalcemia. vi. Chronic COPD creates a respiratory acidosis over time. (a) Gradual destruction of lung tissue inhibits the exchange of oxygen and carbon dioxide. (b) The hypoxic drive is then the only remaining stimulus for breathing. (c) Slow onset of this form of respiratory acidosis makes it survivable. b. Respiratory alkalosis i. Always caused by hyperventilation. (a) Life threatening events may be responsible for hyperventilation. ii. Carbon dioxide levels drop in the blood forcing a reduction in circulating carbonic acid. iii. Renal system begins to retain H+ ions to rebalance the shift (a) Calcium shifts into the intracellular fluid to rebalance the depleted hydrogen levels. iv. The resulting hypocalcemia causes muscle contractions. v. Causes of hyperventilation and respiratory alkalosis include drug overdose (especially aspirin), fever, and overzealous bag-mask ventilation. vi. Signs and symptoms include diminished LOC, light-headedness, carpopedal spasms, tingling lips and face, chest tightness, confusion, vertigo, blurred vision, hypocalcemia, and nausea and vomiting. c. Metabolic acidosis i. Any acidosis not related to the respiratory system ii. Increased respiration is the compensatory mechanism for this condition. (a) Example: Diabetic ketoacidosis and Kussmaul respirations iii. Hydrogen leaks into the cell and serum potassium shifts into the extracellular space, raising the serum potassium level. iv. Calcium also shifts into the extracellular space causing hypercalcemia, which obstructs impulses to neurons in muscles and other tissues. v. Causes include: (a) Lactic acidosis caused by anaerobic cellular respiration (b) Ketoacidosis caused when cells metabolize fatty acids for energy because they are unable to use glucose (1) The by-product of fat metabolism is ketones, compounds that are extremely acidic. (c) Aspirin overdose (1) Aspirin directly stimulates the respiratory center in the brain causing tachypnea, which leads to respiratory alkalosis. (d) Alcohol ingestion leading to alcoholic ketoacidosis (e) Gastrointestinal loses vi. Signs and symptoms include vasodilatation; CNS depression; headaches; hot, flushed skin; hypercalcemia; tachypnea; nausea and vomiting, and cardiac dysrhythmias. d. Metabolic alkalosis i. Occurs when there is excessive loss of acid from increased urine output or decreased acid level in the stomach ii. Common among chronically ill patients iii. Causes include: (a) Upper gastrointestinal losses of acid, such as illness or eating disorders (b) Drinking large amounts of water during vigorous exercise (c) Excessive intake of alkaline substances, such as antacids iv. Compensatory mechanism is the respiratory system (a) Bradypnea develops as a means of retaining carbon dioxide, increasing the level of circulating acid. v. Signs and symptoms include confusion, muscle tremors and cramps, bradypnea, and hypotension. 5. Mixed acid-base imbalance a. Mixed acidosis involves a low pH, an elevated PCO2 level, and a low HCO3– level. i. Occurs when both respiratory and metabolic acidosis are present (a) Common causes are severe trauma, cardiogenic shock, or drug overdose. b. Mixed alkalosis involves an elevated pH, a low PCO2 level, and an elevated HCO3 level. i. May occur when two seemingly unrelated medical issues manifest at the same time (a) Patient may present as uncompensated, partially compensated, or well compensated. VI. Cellular Injury A. Overview 1. Various causes: a. Hypoxia, ischemia, chemical injury, infectious injury, immunologic injury, physical damage, and inflammatory injury b. Manifestations and death depend on how many and which types of cells are damaged. 2. Manifestations occur at both the microscopic and functional levels. a. Common microscopic abnormalities include cell swelling, rupture of cell membranes or nuclear membranes, and breakdown of nuclear material. i. Often results in a change in cell shape and function b. Functional changes may include an inability to use oxygen appropriately, development of intracellular acidosis, accumulation of toxic waste products, and an inability to metabolize nutrients. 3. Damage and functional changes in cells often have an impact on the entire organism. a. The entire organ system may fail. b. Dysfunction in one system inevitably affects other systems. 4. Repair may occur up to a point, with proper treatment. a. Irreversible injury will lead to cell death. b. Cell death is followed by necrosis. i. Process in which the cell breaks down ii. Cell membrane becomes abnormally permeable, and the cell and its organelles swell B. Hypoxic injury 1. Common cause of cellular injury a. Often deadly b. May result from decreased amounts of oxygen in the air or loss of hemoglobin function, a decreased number of red blood cells, disease of the respiratory or cardiovascular system, or loss of cytochromes 2. Cells that are hypoxic for more than a few seconds produce mediators that damage other local or distant body locations. a. A positive feedback cycle leads to more cell damage and more hypoxia. 3. The earliest and most dangerous mediators are free radicals. a. Free radicals: Molecules missing one electron in the outer shell i. Chemical instability causes random attacks on other cells and membranes ii. Results in widespread and potentially deadly tissue damage C. Chemical injury 1. Common poisons: a. Cyanide: Induces cell hypoxia by blocking oxidative phosphorylation in the mitochondria and preventing the metabolism of oxygen b. Pesticides: Block an enzyme preventing proper transmission of nerve impulses 2. Lead a. Chronic ingestion leads to brain injury and neurologic dysfunction. b. Mistaken for calcium in vital biochemical reactions, leading to abnormal results and dysfunction 3. Carbon monoxide a. Binds to hemoglobin i. Prevents adequate oxygenation of the tissues b. Low levels cause nausea, vomiting, and headache. c. Higher levels result in death. 4. Ethanol a. Lower levels cause inebriation. b. Higher doses result in severe CNS depression, hypoventilation, and cardiovascular collapse. 5. Pharmacologic agents a. Produce toxic products when metabolized in the body i. Especially in overdose conditions D. Infectious injury 1. Infectious injury to cells occurs as a result of an invasion of bacteria, fungi, or viruses. 2. Bacteria may cause injury by: a. Direct action on cells b. The production of toxins 3. Viruses often initiate an inflammatory response. a. Leads to cell damage and patient symptoms 4. Virulence a. Measures the disease-causing ability of a microorganism b. Pathogenicity is a function of microorganism’s ability to reproduce and cause disease within the human body. c. Growth and survival of bacteria in the body depend on the: i. Effectiveness of the body’s own defense mechanisms ii. Bacteria’s ability to resist the mechanisms d. Newborn infants, older adults, people with diabetes, and people with cancer or other chronic diseases tend to have weaker immune systems. 5. Bacteria a. Many possess a capsule that protects them from ingestion and destruction. i. Phagocytes: Cells that engulf and consume foreign material b. Categorized depending on results of Gram staining i. Gram staining: Suspension of bacteria is stained with a purple dye and then an iodine solution (a) Then decolorized with alcohol or another solvent (b) Then stained with red dye ii. Bacteria that resist decolorization are called gram-positive bacteria. iii. Bacteria that accept the counterstain are called gram-negative bacteria. c. Produce exotoxins or endotoxins that can injure or destroy cells i. Exotoxins are produced within the cell and released into surrounding tissues or fluids (a) Poisonous (b) Actions vary (c) Inactive exotoxins are sometimes used for vaccines. ii. Endotoxins are lipopolysaccharides that are part of the cell walls of gram-negative bacteria. (a) Cause inflammation, fever, chills, and malaise (b) May cause septic shock if present in large amounts (c) Remain active even after the bacteria are destroyed d. White blood cells are attracted to the site of cell injury. i. Release endogenous pyrogens (cause a fever to develop) e. The body’s most common reaction to the presence of bacteria is inflammation. f. Some bacteria have the ability to produce hypersensitivity reactions. i. Bacteremia: Presence of bacteria in the blood ii. Septicemia: Systematic disease caused by the proliferation of microorganisms 6. Viruses a. Intracellular parasites that take over the metabolic processes of the host cell and use the cell to help them replicate b. Consist of a nucleic acid core of either RNA or DNA c. Capsid: Layer of protein that protects the virus from phagocytosis d. Replication occurs inside the host cell. e. A symbiotic relationship between a virus and normal cells may be the cause of a persistent unapparent infection. E. Immunologic and inflammatory injury 1. Inflammation is a protective response that can occur even without bacterial invasions. a. Infection is characterized by an invasion of microorganisms that causes cell or tissue injury leading to the inflammatory response. i. Can be triggered by an agent that is physical, chemical, or microbiologic b. Local effects include dilatation (expansion) of blood vessels and increased vascular permeability. c. Signs include heat, redness, tenderness, swelling, and pain. d. Severe systemic effects include temperature elevation and increase in the number of leukocytes (leukocytosis). i. The liver produces acute phase proteins that are released in the bloodstream. (a) Help protect the body from the tissue injury e. Outcome depends on amount of tissue damage i. Mild inflammation results in the return of normal tissue. ii. More severe inflammation results in destroyed tissue that must be repaired. (a) Scar tissue replaces large areas of tissue destruction. f. Cellular membranes may be injured when they come in direct contact with the cellular and chemical components of the immune or inflammatory process. i. Potassium leaks out of the damaged cell and water flows inward, causing the cell to swell. (a) Chance of membrane rupture depends on severity of the response g. Immune system protects the body by providing defenses to attack and remove foreign organisms. F. Injurious genetic factors 1. Genetic factors that may damage cells include: a. Chromosomal disorders b. Premature development of atherosclerosis c. Obesity (sometimes) 2. An abnormal gene may develop: a. If the gene mutates during meiosis i. Affects newly formed fetus b. By heredity c. Due to other causes later in life 3. Examples of diseases with a genetic link include Down syndrome and rheumatoid arthritis. G. Injurious nutritional imbalances 1. Good nutrition is required to: a. Maintain good health b. Assist the cells in fighting off disease 2. Injurious nutritional imbalances include: a. Obesity b. Malnutrition c. Vitamin excess or deficiency d. Mineral excess or deficiency 3. These conditions lead to: a. Alterations in physical growth b. Mental and intellectual retardation c. Death H. Injurious physical agents or conditions 1. Physical agents include heat, cold, and radiation a. May cause injury i. Burns, frostbite, radiation sickness, and tumors b. Degree of cell injury is determined by the: i. Strength of the agent ii. Length of exposure I Apoptosis 1. Normal cell death a. Genetically programmed into the cell as a part of normal development, organogenesis, immune function, and tissue growth b. Normal role in aging, early development, menses, lactating breast tissue, thymus involution, and red blood cell turnover 2. During apoptosis: a. Cells exhibit characteristic nuclear changes and die in well-defined clusters. b. Activation of genes that encode for proteins known as caspases essentially leads to cell suicide. c. Controlled degradation allows their remnants to be taken up and reused by neighboring cells. 3. Can be prematurely activated by pathologic factors (cell injury) a. Some forms of heart failure can result in early cell death. b. Death of hepatocytes (liver cells) in patients with viral hepatitis c. Inhibition of normal function allows destructive cellular proliferation (cancer and rheumatoid arthritis). J Abnormal cell death 1. Necrosis: Result of the morphologic changes that occur following cell death in living tissues a. Simple necrosis i. Areas of necrosis where the gross and microscopic tissue and some of the cells are recognizable ii. May be caused by acute ischemia, acute toxicity, or direct physical injury b. Derived necrosis i. Caseation necrosis: Manifested by the loss of all features of the tissue and cells ii. Dry gangrene: Results from invasion and putrefaction of necrotic tissue, after the blood supply is compromised and the tissue undergoes coagulation necrosis iii. Fat necrosis: Results from the destruction of fat cells, usually by enzymes iv. Liquefaction necrosis: Results from coagulation necrosis followed by conversion of tissues into a liquid form and invasion by putrefying bacteria VII. Factors That Cause Disease A. Genetic, environmental, age-related, and sex-associated factors can cause or contribute to disease. 1. Genetic factors are present at birth and are passed through a person’s genes to future generations. 2. Environmental factors include microorganisms, immunologic and toxic exposures, personal habits and lifestyle, exposures to chemicals, the physical environment, and the psychosocial environment. 3. Anatomic causes include malrotation of the colon, degenerative diseases of the spine, or aortic stenosis. 4. Immunological reaction may result in disease. B. Controllable versus uncontrollable risk factors 1. Uncontrollable factors include genetics and race. 2. Controllable factors include smoking, drinking alcohol, inadequate nutrition, lack of physical activity, and stress. 3. Age-related risk a. Risk of a particular disease often depends on the patient’s age. i. Newborns at risk because immune system not fully developed ii. Teenagers at risk because of trauma, use of drugs and alcohol iii. Older adult’s greater risk for cancer, heart disease, stroke, and Alzheimer disease 4. Sex-associated factors a. Some diseases more prevalent in men than women and vice versa i. More prevalent in men: Lung cancer, gout, and Parkinson disease ii. More prevalent in women: Osteoporosis, rheumatoid arthritis, and breast cancer b. Presentation of disease can differ from men to women c. Genetic disorders are related to a person’s sex when the defective gene is located on a sex chromosome. i. Most sex-linked disorders are X linked. (a) May be either recessive or dominant (1) Women with a defective X chromosome may or may not have the disease. (2) Men with a defective X chromosome will always be affected. C. Analysis of disease risk 1. Causal risk factors: Factors that can directly cause a disease to develop 2. Noncausal risk factors: Factors that are associated with risk for a disease but not a direct cause 3. All studies should consider the incidence, prevalence, morbidity, and mortality of the disease. a. Incidence: Number of new cases of a disease in a population b. Prevalence: Number of cases in a particular population within a particular period c. Morbidity: Presence of disease or incidence or prevalence of a disease d. Mortality: Number of deaths from a disease in a given population, expressed as a proportion 4. Risk factors often interact. D. Common familial diseases and associated risk factors 1. Genetic risk: One that is passed through generations by inheritance of a gene 2. Familial tendency: Cluster of diseases in family groups despite lack of evidence for heritable gene-associated abnormalities 3. Autosomal recessive: Pattern of inheritance that involves genes located on autosomes (a person needs to inherit two copies of a particular gene). 4. Autosomal dominant: A person needs to inherit only one copy of a particular form of a gene to show that trait. 5. Immunologic disorders a. Caused by either hyperactivity or hypoactivity of the immune system b. Most that exhibit familial tendencies involve an overactive immune system (allergies, asthma, and rheumatic fever) c. Allergies: Acquired following initial exposure to a stimulant (allergen); repeated exposures cause the immune system to react. i. Person who has an allergic tendency is atopic d. Asthma: Chronic inflammatory condition resulting in intermittent wheezing and excess mucus production 6. Cancer a. Includes a large number of malignant growths b. Prognosis often depends on the extent of its spread (metastasis) and the effectiveness of treatment. c. Lung cancer: Leading cause of death due to cancer in the United States i. Major risk factor is cigarette smoking ii. Eight alterations in genetic material of lung cancer that may suggest a genetic tendency (a) Other predisposing factors include exposure to asbestos, coal products, and other industrial and chemical products. iii. Symptoms include cough, difficulty breathing, blood tinged sputum, and repeated infections. iv. Treatment depends on type, site, and extent. (a) May include surgery, chemotherapy, and/or radiotherapy d. Breast cancer is the most common type of cancer occurring among women. i. Women whose first-degree relatives have breast cancer are 2.1 times more likely to develop the disease. ii. Risk varies with the age at which the affected relative was diagnosed. iii. 5 to 10% of patients have a pattern of autosomal dominant inheritance iv. Symptoms: (a) Early: Small painless lump, thick or dimpled skin, change in nipple (b) Late: Nipple discharge, pain and swollen lymph glands in the axilla v. Treatment depends on location, size, and metastasis of the tumor e. Colorectal cancer: Third most common type of cancer in both males and females i. Relatives of patients with colorectal cancer are more likely to have the disease themselves. ii. Symptoms: May be minimal (small amounts of blood in the stool) iii. Treatment involves surgery and sometimes chemotherapy. 7. Endocrine disorders a. Diabetes mellitus: Chronic disorder of metabolism associated with either partial insulin secretion or total lack of insulin secretion by the pancreas i. Symptoms include excessive thirst and urination, weight abnormalities, and the presence of excessive glucose in the urine and blood. b. Ketoacidosis-prone (type 1) diabetes: Insulin-dependent diabetes mellitus c. Non-ketoacidosis-prone (type 2) diabetes: Non-insulin-dependent diabetes d. No cure for type 1; type 2 can occasionally be brought under control with weight loss and medications. 8. Hematologic disorders a. Hemolytic anemia: Characterized by increased destruction of red blood cells i. Number of causes: Rh factor blood transfusion reaction, disorder of the immune system, exposure to bacterial toxins or chemicals b. Hemophilia: Inherited disorder characterized by excessive bleeding i. Sex-linked condition (occurring only in males), passed from asymptomatic mothers to sons ii. One of the blood-clotting proteins is missing or is present in abnormally low amounts. c. Hemochromatosis: Inherited (autosomal recessive) disease in which the body absorbs more iron than it needs i. Excess iron is stored in various organs. ii. Can lead to diabetes, heart disease, liver disease, arthritis, impotence, and a bronzed skin color. 9. Cardiovascular disorders a. Long QT syndrome: Cardiac conduction system abnormality resulting in a prolongation of the QT interval on the ECG i. Inherited in an autosomal dominant manner ii. Sometimes associated with congenital hearing loss, hypertrophic cardiomyopathy, or mitral valve prolapse iii. Most patients are asymptomatic until they have a dysrhythmia causing syncope or sudden death. iv. Consider syncope to be due to a life threat until proven otherwise if one of the following conditions: (a) Exercise-induced syncope (b) Syncope associated with chest pain (c) History of syncope in a close family member (d) Syncope associated with startle b. Cardiomyopathy: Diseases of the myocardium that ultimately progress to heart failure, acute myocardial infarction, or death i. Heart muscle becomes thin, flabby, dilated, or enlarged. ii. Hypertrophic cardiomyopathy is genetically autosomal dominant. (a) Excessive thickening of the heart muscle (b) Symptoms may include shortness of breath, chest pain, palpations, or syncope. (c) Treatment includes beta blockers in some cases, or surgery or an autonomic implantable cardiac defibrillator. c. Mitral valve prolapse: Mitral valve leaflets balloon into the left atrium during systole. i. Often benign and asymptomatic (a) Only physical finding may be a “clicking” sound heard during cardiac auscultation. ii. May lead to a mitral regurgitation (a) Large amount of blood leaks backward through the defective valve. (b) Can lead to thickening or enlargement of the heart wall (c) Patients feel tired or short of breath. (d) Treatment can be medication and in some cases surgery to repair or replace the valve. d. Coronary heart disease: Caused by impaired circulation to the heart i. Typically from occluded coronary arteries from atherosclerotic plaque buildup ii. Familial tendency (a) Significant risk factors include having a father who had an acute myocardial infarction or died suddenly before 55 years of age or having a mother who died before 65 years of age. iii. Other risk factors include hypercholesterolemia, cigarette smoking, hypertension, age, and diabetes. iv. Hypercholesterolemia: Elevation of the blood cholesterol level e. Hypertension: Elevated blood pressure i. Associated with an increased risk of coronary artery disease and stroke ii. Risk factors can be genetic or lifestyle related. (a) Include age, race, sex, family history, obesity or being overweight, sedentary lifestyle, tobacco use, diet, and stress iii. Stroke risk factors are also genetic or lifestyle related. (a) Include age (55 or older), race (African Americans, Hispanics, and American Indian/Alaska Natives), sex (men), family history, obesity or being overweight, sedentary lifestyle, hypertension, hypercholesterolemia, cigarette smoking, diabetes, cardiovascular disease, use of birth control pills or hormone therapy, and excessive alcohol consumption. 10. Renal disorders a. Gout: Abnormal accumulation of uric acid due to a defect in metabolism i. Accumulates in the blood and joints, causing pain and swelling ii. More common among men than women and usually has a genetic basis iii. Left untreated, causes destructive tissue changes in the joints and kidneys iv. Treatment includes diet and drugs. b. Kidney stones: Small masses of uric acid or calcium salts that form in any part of the urinary system i. Often cause severe pain, nausea, and vomiting when the body attempts to pass them ii. Researchers have found a gene that causes the intestines to absorb too much calcium. iii. Uric acid stones often have a genetic basis. 11. Gastrointestinal disorders a. Malabsorption disorders: Result from defects in the function of the bowel wall that prevents normal absorption of nutrients i. Complex symptoms include loss of appetite, bloating, weight loss, muscle pain, stools with high fat content, and diarrhea that may be bloody. ii. Lactose intolerance: Defect in the enzyme lactase (a) Inability to digest lactose (milk sugar) (b) Symptoms include bloating, flatulence, abdominal discomfort, nausea, and diarrhea on ingestion of milk and milk products. iii. Ulcerative colitis: Serious chronic inflammatory disease of the large intestine and rectum (a) Shows a familial tendency (b) Recurrent episodes of abdominal pain, fever, chills, and profuse diarrhea, with stools containing pus, blood, and mucus (c) Treatment consists of anti-inflammatory agents and in some cases surgery. iv. Crohn’s disease: Chronic inflammatory condition affecting the colon and/or terminal part of the small intestine (a) Believed to be associated with as-yet-undetermined gene abnormalities (b) Symptoms include frequent episodes of diarrhea, melena, abdominal pain, nausea, fever, weakness, and weight loss. (c) Management includes anti-inflammatory agents, antibiotics, proper nutrition, and sometimes surgery. b. Peptic ulcer disease: Circumscribed erosions (ulcerations) in the mucous membrane lining of the gastrointestinal tract (specifically esophagus, stomach, duodenum, or jejunum) i. May be associated with excess acid production or from a breakdown in the normal mechanisms protecting the mucous membranes ii. Appears to have a genetic component, but major contributor is infection with the bacterium Helicobacter pylori iii. Symptoms include gnawing pain (often worsened when the stomach is empty, after the person eats certain foods, or when the patient is under stress). iv. Treatment includes avoidance of irritants, antibiotics, and drugs to decrease acidity. c. Gallstones (choleliths): Stone-like masses in the gallbladder or its ducts caused by precipitation of substances contained in bile i. Contributing factors include abnormalities in the composition of bile, or stasis of bile. ii. May be asymptomatic or cause inflammation of the gallbladder, pain in upper right quadrant or epigastric area, or jaundice d. Obesity: Unhealthy accumulation of body fat i. Body mass index of greater than or equal to 30 kg/m2 ii. Decreases life span by average of 8 to 13 years iii. Morbid obesity: Body mass index of greater than or equal to 40 kg/m2 (a) Includes all the health risks of obesity but makes basic life functions such as walking or breathing difficult. iv. Overweight: Body mass index of 25 to 29.9 kg/m2 v. Health risks include hypertension, hyperlipidemia, cardiovascular disease, glucose intolerance, insulin resistance, diabetes, gallbladder disease, infertility, and cancer of the endometrium, breast, prostate, and colon. vi. Social and psychological effects include depression, anxiety, shame, rejection, and discrimination. vii. Although there may be a genetic predisposition to obesity, behavioral and environmental factors are better known. (a) Including a sedentary lifestyle, overeating, or choosing foods high in calories and low in nutritional value 12. Neuromuscular disorders a. Huntington’s disease: Hereditary condition characterized by progressive chorea and mental deterioration, leading to dementia i. Symptoms usually first appear in the third or fourth decade and progress to death often within 15 years. b. Muscular dystrophy: Generic term for a group of hereditary diseases of the muscular system i. Characterized by weakness and wasting groups of skeletal muscles, leading to increasing disability ii. Various forms differ in age of onset, rate of progression, and mode of genetic transmission. iii. Duchenne muscular dystrophy: Sex-linked recessive disease (affecting only males) (a) Symptoms first appear around the age of 4. (b) Progressive wasting of leg and pelvic muscles producing a waddling gait and abnormal curvature of the spine, progressing to inability to walk and confinement to a wheelchair (c) No known treatment c. Multiple sclerosis: Progressive disease in which nerve fibers of the brain and spinal cord lose their myelin cover i. Not directly inherited, but patients may have familial predisposition ii. Usually begins in early adulthood and progresses slowly with periods of remission and exacerbation iii. Symptoms include abnormal sensations in the face or extremities, weakness, visual disturbances, ataxia, abnormal reflexes, tremors, difficulty urinating, and difficulty walking. iv. No specific treatment or cure d. Alzheimer’s disease: Unknown cause i. Results in cortical atrophy; loss of neurons in the frontal and temporal lobes of the brain; and ventricular enlargement due to loss of brain tissue ii. Studies of the genetics of inherited early-onset Alzheimer’s have been linked to mutations on three genes. iii. Progressive disease (a) Early symptoms include memory loss, lack of spontaneity, subtle personality changes, and disorientation to time and date. (b) Advanced symptoms include indifference to food, inability to communicate, urinary and fecal incontinence, and seizures 13. Psychiatric disorders a. Schizophrenia: Group of mental disorders characterized by gross distortions of reality (psychoses), withdrawal from social contacts, and disturbances of thought, language, perception, and emotional response i. Symptoms are highly varied. ii. No identified cause (a) Combination of hereditary or genetic predisposing factors is likely b. Bipolar disorder: Mental disorder characterized by episodes of mania and depression i. Evidence of role of genetics ii. Treatment consists of psychotherapy plus antidepressants and tranquilizers. VIII. Hypoperfusion A. Perfusion is the delivery of oxygen and nutrients and removal of wastes from the cells, organs, and tissues by the circulatory system. B. Hypoperfusion occurs when the level of tissue perfusion decreases below normal. 1. When the body senses tissue hypoperfusion, it sets compensatory mechanisms into motion. a. Sometimes sufficient to stabilize the patient b. Other times it overwhelms the normal compensatory mechanisms, and the patient’s condition deteriorates. 2. Response to hypoperfusion a. The body releases catecholamines (epinephrine and norepinephrine), which produce vasoconstriction. b. The RAAS is activated, and antidiuretic hormone is released from the pituitary gland. c. Actions trigger salt and water retention as well as peripheral vasoconstriction, increasing blood pressure and cardiac output. d. Fluid shifts from the interstitial tissues into the vascular compartment. 3. The overall response is to increase the preload, stroke volume, and heart rate so that blood volume is adequate. a. Result is often an increase in cardiac output and myocardial oxygen demand 4. Persistence results in a continued increase in myocardial oxygen demand. a. Compensatory mechanisms can no longer keep up with the demand. i. Myocardial function worsens. ii. Tissue perfusion decreases, causing impaired cell metabolism. iii. Fluid may leak from the blood vessels causing systemic and pulmonary edema. iv. Other signs may be present: dyspnea, dusky skin, low blood pressure, oliguria, and impaired mentation IX. Types of Shock A. Shock is an abnormal state associated with inadequate oxygen and nutrient delivery to the metabolic apparatus of the cell 1. Results in impairment of cell metabolism and inadequate perfusion of vital organs a. Prevents the body from properly using oxygen and glucose at the cellular level i. Cells revert to anaerobic metabolism. (a) Causes increased lactic acid production and metabolic acidosis, decreased oxygen affinity for hemoglobin, decreased ATP production, changes in electrolytes, cellular edema, and release of lysosomal enzymes ii. Glucose impairment leads to elevated blood glucose levels due to release of catecholamines and cortisol. 2. Occurs due to inadequacy of the central circulation or of the peripheral circulation a. Central shock: Consists of cardiogenic shock and obstructive shock b. Peripheral shock: Includes hypovolemic shock and distributive shock B. Central shock 1. Cardiogenic shock a. Occurs when the heart cannot circulate enough blood to maintain adequate peripheral oxygen delivery b. Most common cause is myocardial infarction 2. Obstructive shock a. Occurs when blood flow becomes blocked in the heart or great vessels i. Pericardial tamponade: Diastolic filling of the right ventricle is impaired due to significant amounts of fluid in the pericardial sac surrounding the heart. ii. Aortic dissection leads to a false lumen with loss of normal blood flow. iii. Left atrial tumor may obstruct flow between the atrium and ventricle and decrease cardiac output iv. Obstruction of either the superior or inferior vena cava decreases cardiac output by decreasing venous return. v. A large pulmonary embolus or a tension pneumothorax may prevent adequate blood flow to the lungs, resulting in inadequate venous return to the left side of the heart. C. Peripheral shock 1. Hypovolemic shock a. Circulating blood volume is unable to deliver adequate oxygen and nutrients to the body. b. Two types: Exogenous and endogenous i. Depends on where the fluid loss occurs c. External bleeding: Most common type of exogenous hypovolemic shock d Endogenous shock occurs when the fluid loss is contained within the body. 2. Distributive shock a. Occurs when there is widespread dilation of the resistance vessels, the capacitance vessels, or both i. Circulating blood volume “pools” in the expanded vascular beds, and tissue perfusion decreases. b. Three common types: i. Anaphylactic shock (anaphylaxis) (a) Histamine and other vasodilator proteins are released upon exposure to an allergen. (b) Accompanied by wheezing and urticaria (hives) (c) Widespread vasodilatation causes fluid to leak out of the blood vessels into the interstitial spaces ii. Septic shock (a) Result of widespread infection (b) If normal immune mechanisms become overwhelmed, the body produces a multitude of substances that cause vasodilation and decreased cardiac output. (c) If untreated, results in multiple organ dysfunction syndrome and death iii. Neurogenic shock (a) Usually results from spinal cord injury (b) Loss of normal sympathetic nervous system tone and vasodilation D. Management of shock 1. Most types of shock are characterized by reduced cardiac output, circulatory insufficiency, and rapid heartbeat. 2. Determining the presence or absence of shock requires the evaluation of the presence and volume of the peripheral pulses, and assessment of end-organ perfusion and function. a. The strength of peripheral pulses is related to both stroke volume of the heart and pulse pressures. b. Normal skin perfusion results in warm, dry, and pink extremities, fingers, and toes. c. Shock may result in slow, delayed, or prolonged capillary refill time. i. To test, briefly squeeze the toenail or fingernail and look for return color. (a) Normal refill time is less than 2 seconds. 3. Mottling, pallor, peripheral or central cyanosis, and delayed capillary refill may signal the presence of shock. 4. Accuracy of capillary refill measurement decreases after age 6. a. Not as useful in the adult population X. Multiple Organ Dysfunction Syndrome (MODS) A. MODS is a progressive condition that occurs in some critically ill patients. 1. Characterized by concurrent failure of two or more organs or organ systems that were initially unharmed a. Six organ systems surveyed in diagnosing: Respiratory, hepatic, renal, hematologic, neurologic, and cardiovascular b. Overall mortality rate 60 to 90%, but condition is often reversible c. Major cause of death following sepsis, trauma, and burn injuries 2. Types include: a. Primary MODS: Direct result of an insult b. Secondary MODS: Slower more progressive organ dysfunction 3. Results when injury or infection triggers a massive systemic immune, inflammatory, and coagulation response accompanied by endotoxin release a. Overactivation of the complement system further increases inflammation and damage to the cells. b. Overactivation of the coagulation system causes uncontrolled coagulation in the microscopic venules and arterioles c. Bradykinin, a potent vasodilator, is released. i. Vasodilation leads to tissue hypoperfusion and may contribute to hypotension. 4. Net outcome is maldistribution of systemic and organ blood flow 5. Typically develops hours to days following resuscitation 6. Signs and symptoms include hypotension, insufficient tissue perfusion, uncontrollable bleeding, and multisystem organ failure. a. During 14- to 21-day period, renal and liver failure can develop, along with collapse of the gastrointestinal and immune systems. i. Patients may require continuous bedside dialysis. ii. No definitive treatment for liver failure. b. Brain, adrenal glands, and heart are affected early on. i. Hypotension cannot be controlled. ii. Cardiovascular collapse and death typically occur within days to weeks of the initial insult. XI. The Body’s Self-Defense Mechanisms A. The immune system includes all structures and processes associated with the body’s defense against foreign substances and disease-causing agents. 1. Three lines of defense: Anatomic barriers, the immune response, and the inflammatory response B. Anatomic barriers 1. Several decrease the chances of bodily invasion by foreign substances. a. Skin b. Hairs in the upper respiratory tract and the lining of the lower respiratory tract c. Acid in the stomach C. Immune response 1. Body’s defense reaction to any substance that is recognized as foreign a. Directed toward invading microbes (bacteria or viruses) b. Also triggered by foreign bodies and even abnormal growths in the cells 2. Involves only one type of white blood cells (lymphocytes) a. Not all invaders can be destroyed. b. Often succeeds in preventing severe disease following infection 3. Anatomy of the immune response a. Lymphatic system i. Network of capillaries, vessels, ducts, nodes, and organs that helps maintain the fluid environment of the body ii. Produces lymph and conveys it through the body b. Two anatomic components: Lymphoid tissues and cells responsible for immune response c. Lymphoid tissues i. Distributed throughout the body ii. Two primary tissues: Bone marrow and thymus gland (a) Bone marrow: Specialized soft tissue found within bone (1) T lymphocytes originate from precursor cells in the bone marrow, leave the bone marrow, and mature in the thymus gland. (b) Thymus gland: Bilobed gland that enlarges until puberty, when it then begins to shrink and decrease in functional activity iii. In secondary lymphoid tissues (encapsulated and unencapsulated), mature immune cells interact with invaders and initiate a response. (a) Encapsulated lymphoid tissues consist of: (1) Lymph nodes: Small structures that filter lymph and store lymphocytes (2) Spleen: Monitors the blood, destroys worn-out red blood cells, and traps foreign invaders iv. Lymph: Thin watery fluid that bathes the tissues of the body (a) Circulates through the lymph vessels and is filtered in lymph nodes v. Clusters of lymphoid tissue are collectively termed mucosal-associated lymphoid tissue. (a) Contain immune cells that are in a position to intercept pathogens before they reach the general circulation (b) Tonsils: Best known example vi. Gut-associated lymphoid tissue (GALT): Unencapsulated lymphoid tissue in the gastrointestinal tract d. Salivary glands and lacrimal glands produce an antibody (secretory immunoglobulin A). i. Saliva fights pathogens that enter the mouth. ii. Tears fight pathogens that enter the eye. e. Leukocytes (white blood cells): Primary cells of the immune system i. Basophils: Contain histamine granules and other substances that are released during inflammatory and allergic responses ii. Eosinophils: Release substances that damage or kill parasitic invaders and play a major role in mediating the allergic response iii. Neutrophils: Most abundant white blood cells (a) Largely responsible for protecting the body against infection (b) Key components of the first response to foreign body invasion (c) Phagocytosis: Process in which neutrophils attract foreign antigens and then destroy them by engulfing and digesting them iv. Monocytes: Travel to tissues and differentiate into macrophages (scavengers for the tissues) v. Lymphocytes: Mediate the acquired immune response (a) Two basic types: B lymphocytes and T lymphocytes f. Mast cells: Resemble basophils but are found in connective tissues, beneath the skin, in the gastrointestinal mucosa, and in the mucosal membranes of the respiratory system 4. Characteristics of the immune response a. Native and acquired immunity protect the body from infectious agents and foreign substances b. Natural (native) immunity: Nonspecific cellular and humoral response that operates as the first line of defense against pathogens i. Associated with the initial inflammatory response c. Acquired (adaptive) immunity: Highly specific method by which cells respond to an immune stimulant i. Arises when the body is exposed to a foreign substance or disease and produces antibodies to that invader ii. Passively acquired immunity is the receipt of preformed antibodies to fight or prevent infection. (a) Example: Passage of antibodies from mother to infant through milk; injection of immunoglobin d. Primary (initial) immune response takes place during first exposure to an antigen (foreign substance). i. Secondary immune response occurs on repeat exposure to a foreign substance. e. Beginning (induction) phase of the immune response occurs when immune system recognizes an antigen. i. Antigens may be immunogenic (elicit response) or nonimmunogenic (do not elicit response). ii. Antibody: Binds a specific antigen so the complex can attach itself to specialized immune cells that destroy the complex or induce a response iii. Immunogen: An antigen capable of generating an immune response against itself (a) Hapten: Substance that normally does not stimulate an immune response but that can be combined with an antigen and at a later time initiate a specific antibody response on its own 5. Humoral immune response a. Humoral immunity: B-cell lymphocytes produce antibodies (immunoglobulins) that react with a specific antigen. b. B lymphocytes are born in the bone marrow (descended from stem cells). i. Clonal selection theory: Each B cell makes antibodies that have only one type of antigen-binding region. (a) Specific for a particular antigen (cognate antigen) ii. For B cells to produce antibodies, they must be activated. (a) Most common way this occurs is via helper T cells. (1) Macrophage engulfs the antigen via phagocytosis, pushing discarded particles to the cell surface where they interact with B cells and a helper T cell (2) Antigen binds to the B cell and the helper T cell, activating both (3) Helper T cells secrete a lymphokine, which stimulates the B cell to produce a clone (group of identical cells formed from the same parent cell). (4) Clone has two identical cells that have two different functions; plasma cells make antibodies and memory cells remember the encounter with antigen iii. Major histocompatibility complex: Group of genes located on a single chromosome that distinguishes between foreign substances and its own cells and tissues iv. Human leukocyte antigen gene complex: The human major histocompatibility complex that encodes for numerous antigens that are unique to a person 6. Immunoglobins are antibodies secreted by B cells. a. Consist of a crystallizable fragment portion and two antigen-binding fragment regions i. Bind only a specific antigen b. The basic antibody molecule has four chains linked into a Y shape. i. Each side is identical with one light chain attached to a heavy chain. c. Three main categories of antigens on antibodies i. Isotypic antigenic marker occurs in all members of a subclass of an immunoglobin class ii. Allotypic antigenic marker found on some members of a subclass immunoglobin class iii. Idiotypic antigenic determinant is unique structure created on the light and heavy chain of immunoglobulin molecule d. Antibodies make antigens more visible to the immune system in three ways: i. Opsonization: An antibody coats an antigen to facilitate its recognition by immune cells ii. Antibodies cause antigens to clump for easier phagocytosis. iii. Antibodies bind to and inactivate some toxins produced by bacteria so macrophages can ingest them. e. Antibodies divided into five general classes of immunoglobulins i. IgG accounts for 75% of the antibodies in the blood. ii. IgA accounts for 15% of the antibodies in the blood. iii. IgM accounts for 5 to 10% of the antibodies in the blood. iv. IgE accounts for less than 1% of the antibodies in the blood. v. IgD accounts for less than 1% of the antibodies in the blood. f. Fetal immunity: Passively acquired immunity derived from maternal IgG and IgM antibodies i. Immunity passes through trophoblast (group of cells formed from the ovum’s peripheral cells). 7. Cell-mediated immune response a. Cell-mediated immunity: Characterized by the formation of a population of lymphocytes that can attack and destroy foreign material b. Main defense against virus, fungi, parasites, and some bacteria c. Mechanism by which the body rejects transplanted organs and eliminates abnormal cells that sometimes arise d. T-cell lymphocytes recognize antigens and contribute to the immune response in two ways. i. Secrete cytokines that attract other cells ii. Become cytotoxic and kill infected or abnormal cells e. Five subcategories of T cells i. Killer T cells: Destroy the antigen. ii. Helper T cells: Activate immune cells, including B cells and other T cells. iii. Suppressor T cells: Suppress the activity of other lymphocytes so they do not destroy normal tissue. iv. Memory T cells: Remember the reaction for the next time it is needed. v. Lymphokine-producing cells: Work to damage or destroy cells infected with a virus. 8. Cellular interactions in the immune response a. Depends on the particular challenge, but the basic pattern is the same. i. Bacteria enters the body. (a) If they are not encapsulated, macrophages begin to ingest them. (b) If they are encapsulated, antibodies coat the capsule so they can be ingested by phagocytes. ii. Components of the cell wall activate the complement system. (a) Chemotaxins attract leukocytes from the circulation to help fight the infection. iii. Complement cascade ends with formation of membrane attack complex (a) Molecules insert themselves into the bacterial membrane, weakening the membrane and allowing water to enter. iv. Already existing antibodies will assist by acting as opsonins and neutralizing the bacterial toxins. v. Memory B cells attracted to the infected site will be activated if they encounter an antigen they recognize. vi. If infection is new to the body, B cells will be activated. D. Inflammatory response 1. Response of the tissues of the body to irritation or injury characterized by pain, swelling, redness, and heat a. White blood cells are a major component of this response. b. Inflammatory reaction and immune response are independent processes although they often occur simultaneously. c. Two most common causes: Infection and injury 2. Acute inflammation a. Involves both vascular and cellular components i. Active hyperemia (influx of blood under increased pressure) causes blood vessel to expand. ii. Vessel wall becomes thinner and fluid leaks into interstitial spaces (edema). iii. When enough pressure has been released, vessel wall contracts and outflow slows (a) Leads to stasis of blood in the capillaries 3. A variety of blood cells participate: a. White blood cells (leukocytes) b. Platelets c. Mast cells d. Plasma cells (B lymphocytes) e. Specific cell types include neutrophils, monocytes, lymphocytes, eosinophils, basophils, and activated platelets. f. Chemical mediators account for the vascular and cellular events that occur. i. Include histamine, arachidonic acid derivatives, and cytokines 4. Mast cells a. Degranulate and release a variety of substances i. Major stimuli for degranulation are physical injury, chemical agents, and immunologic substances. b. Release vasoactive amines i. Histamine and serotonin: Increase vascular permeability, cause vasodilation, and can cause bronchoconstriction, nausea, and vomiting c. Synthesize leukotrienes (slow-reacting substances of anaphylaxis) i. Family of biologically active compounds derived from arachidonic acid ii. Participate in host defense reactions and pathophysiologic conditions (immediate hypersensitivity and inflammation) iii. Primarily endogenous mediators of inflammation iv. Contribute to the signs and symptoms seen in acute inflammatory responses d. Synthesize prostaglandins i. Substances derived from arachidonic acid ii. Comprise a group of about 20 lipids that are modified fatty acids attached to a five-member ring iii. Found in many vertebrate tissues iv. Act as messengers involved in reproduction, the inflammatory response to infection, and pain 5. Plasma protein systems: Plasma-derived mediators that modulate the inflammatory response a. Complement system i. Group of plasma proteins that attract white blood cells to sites of inflammation, activate white blood cells, and directly destroy cells ii. C3: Central compound, produced by one of the two pathways (a) Classic pathway: Starts when an antigen-antibody complex binds to a complement component (1) Activation is dependent on the presence of antibodies. (b) Alternate pathway: Triggered by bacterial toxins and does not need antibodies to be activated iii. Components include: (a) C3b: Coats bacteria, making it easier for macrophages to engulf them (b) Anaphylatoxins (C3a, C4a, and C5a): Stimulate smooth-muscle contraction and increase vascular permeability (c) Membrane attack complex (C5b, C6, C7, C8, and C9): Bind to form a hollow tube that can puncture into the plasma membrane of a cell b. Coagulation system i. Performs a vital role in the formation of blood clots and facilitates repairs to the vascular tree ii. Inflammation triggers a series of reactions that encourage fibrin formation. (a) Fibrin: Protein that polymerizes (bonds) to form the fibrous component of a blood clot (b) Fibrinolysis cascade: Activated to dissolve the fibrin and create fibrin split products c. Kinin system i. Leads to the formation of the vasoactive protein bradykinin from kallikrein (a) Kallikrein: Enzyme normally found in blood plasma, urine, and body tissue in an inactive state (b) Bradykinin: Increases vascular permeability, dilates blood vessels, contracts smooth muscle, and causes pain when injected into the skin ii. Hageman factor: Spurs the kinin system into action (a) Triggers intrinsic clotting cascade, which occurs when blood is exposed to collagen or other substances 6. Cellular components of inflammation a. Goal: For inflammatory cells (specifically polymorphonuclear neutrophils [PMNs]) to arrive at the sites within tissue where they are needed b. Involves two major stages: i. Intravascular phase: Leukocytes move to the sides of blood vessels and attach to the endothelial cells. ii Extravascular phase: Leukocytes travel to the site of inflammation and kill organisms. c. Cellular event sequence i. Margination: Loss of fluid causes blood to increase in viscosity (a) Blood flow slows and produces stasis. (b) Leukocytes move toward the sides of blood vessels and bind to endothelial cells. ii. Activation: Mediators trigger the appearance of selectins and integrins on the surfaces of endothelial cells and PMNs. iii. Adhesion: PMNs attach to endothelial cells. iv. Transmigration (diapedesis): PMNs permeate through the vessel wall, moving into interstitial space. v. Chemotaxis: PMNs move toward the site of inflammation in response to chemotactic factors. 7. Cellular products of inflammation a. Cytokines: Products of cells that affect the function of other cells i. Interleukins (include IL-1 and IL-2): Attract white blood cells to the sites of injury and bacterial invasion ii. Interferon: Protein produced by cells when they are invaded by viruses (a) Released into the bloodstream or intercellular fluid to induce healthy cells to manufacture an enzyme that counters the infection b. Lymphokines: Stimulate leukocytes i. Macrophage-activating factor stimulates macrophages to engulf and destroy foreign substances. ii. Migration inhibitory factor keeps white blood cells at the site of infection or injury until they can perform their designated task. 8. Injury resolution and repair a. Normal wound healing involves four steps: i. Repair of damaged tissues ii. Removal of inflammatory debris iii. Restoration of tissues to a normal state iv. Regeneration of cells b Healing after injury or loss depends on the type of cells that make up the affected organ. i. Labile cells: Divide continuously so organs derived from these cells heal completely. ii. Stable cells: Replaced by regeneration from remaining cells iii. Permanent cells: Cannot be replaced; scar tissue is laid down instead c. Wounds can heal by: i. Primary intention: Occurs in clean wounds with opposed margins ii. Secondary intention: Occurs in large gaping or infected wounds (a) More pronounced and prolonged inflammatory phase 9. Dysfunctional wound healing a. Factors that lead to dysfunctional wound healing may be local or systemic i. Local factors include infection, an inadequate blood supply, and foreign bodies. ii. Systemic factors include poor nutritional intake and hematologic abnormalities. (a) Diabetes and AIDS affect the cells of the immune system, increasing the chance for infections. (b) Corticosteroids suppress the initial inflammatory response required for the proper formation of scar tissue and increase the risk of wound infection. (c) Wound separation slows down the healing process as healing needs to start over to some extent. E. Chronic inflammatory responses 1. Causes include: a. Unsuccessful acute inflammatory response to a foreign body b. Persistent infection c. Presence of an antigen 2. Associated with an infiltrate (pus) containing monocytes and lymphocytes 3. Usually involve tissue destruction and repair 4. Events are similar to those in acute inflammation process but also include the growth of new blood vessels (angiogenesis) XII. Variances in Immunity and Inflammation A. Hypersensitivity is any response of the body to any substance to which a patient has increased sensitivity. 1. Allergy: Hypersensitivity reaction to the presence of an agent (allergen) 2. Autoimmunity: Production of antibodies or T cells that work against the tissues of one’s own body 3. Isoimmunity: Formation of T cells or antibodies directed against the antigens on another person’s cells a. Typically after organ transplant or blood transfusion 4. Transient neonatal diseases: Present at birth but eventually resolve a. Examples include transient neonatal hyperglycemia, transient neonatal myasthenia gravis, and transient neonatal neutropenia. b. Occur due to pathogenic immunoglobulins passing from the pregnant woman to the fetus 5. Hypersensitivity reactions may be immediate or delayed, depending on the antigen and the type of response the body mounts against it. a. Type I: Immediate hypersensitivity reaction i. Acute reaction that occurs in response to a stimulus ii. Involves interaction between the stimulus and a preformed antibody of the IgE type iii. Severity of symptoms depends on the extent of mediator release (a) Varies from life threatening to milder reactions iv. Propensity may be diagnosed through skin tests and laboratory procedures v. Treatment in the field includes administration of epinephrine via EpiPen auto-injector or subcutaneous injection. b. Type II: Cytotoxic hypersensitivity i. Involves the combination of IgG or IgM antibodies with antigens on the cell membrane ii. Cells are destroyed by complement fixation or by other antibodies. (a) Healthy cells may be destroyed. iii. Occurs within a few hours of exposure iv. Examples include blood transfusions. c. Type III: Tissue injury caused by immune complexes i. Involves IgG antibodies that form immune complexes with the antigen to recruit phagocytic cells to a site where they can release inflammatory cytokines ii. Occurs within a few hours of exposure iii. Reactions may be systemic or localized. (a) Systemic form (serum sickness) results from a large, single exposure to an antigen (b) Localized form (Arthus reaction) consist of a circumscribed area of vascular inflammation (vasculitis) d. Type IV: Delayed (cell-mediated) hypersensitivity i. Primarily mediated by soluble molecules that are released by specifically activated T cells ii. Subtypes: (a) Delayed hypersensitivity involves lymphocytes and macrophages (b) Cell-mediated cytotoxicity involves sensitized T cells (T killer cells) 6. Targets of hypersensitivity reactions a. Allergic reactions: Target is an antigen or allergen b. Autoimmune reactions: Target is person’s own tissue i. Graves’ disease: Autoimmune disease caused by thyroid-stimulating or thyroid-growth immunoglobulins ii. Type 1 diabetes mellitus: Some agent causes the body to produce autoantibodies against beta cells. iii. Rheumatoid arthritis: Chronic systemic disease (a) One of the most common forms is characterized by inflammation of the synovium (connective tissue membrane lining the joint). (b) Inflammatory cells release enzymes that cause damage to bone and cartilage. iv. Myasthenia gravis: Acquired autoimmune disease characterized by attack on the nerve-muscle junction v. Neutropenia: Decrease in circulating neutrophils (a) Decreases the body’s ability to fight infection vi. Immune thrombocytopenia purpura (ITP): Blood disorder in which the patient forms antibodies to blood platelets that cause their destruction (a) Bleeding is the main symptom. (b) Treatment is based on severity of symptoms and platelet count. vii. Systemic lupus erythematosus (SLE): Body’s own immune system is directed against the body’s own tissues c. Blood group antigens i. Rh factor: Antigen present in the erythrocytes of about 85% of the population ii. Blood type is determined by presence or absence of specific antigens. (a) Type A contains erythrocytes with type A surface antigens. (b) Type B contains type B surface antigens and plasma containing type A antibodies. (c) Type AB contains both types of antigens but the plasma contains no ABO antibodies. (d) Type O contains neither A nor B antigens but contains both A and B plasma antibodies. iii. Persons who have any of the 18 separate Rh antigens are designated Rh-positive. B. Immune deficiencies 1. Immunodeficiency: Abnormal condition in which some part of the body’s immune system is inadequate 2. Congenital immunodeficiencies a. Defects involve lymphoid stem cells and affects both T and B cells b. Both forms of this disease are inherited. i. X-linked agammaglobulinemia: Affects male infants and is caused by a defect in the differentiation of pre-B cells into B cells (a) Results in markedly decreased levels of all immunoglobulins and mature B lymphocytes ii. Isolated deficiency of IgA: Results from a block in the terminal differentiation of B lymphocytes. (a) Most patients are asymptomatic, but can have chronic sinus infections 3. Acquired immunodeficiencies a. Contributors to this condition include: i. Nutritional deficiency ii. Stress of trauma iii. Hypoperfusion or shock iv. Mediator production v. Damage to vital organs vi. Decreased nutrition occurring during trauma states b. Iatrogenic (treatment-induced) immunodeficiency: most frequently caused by drugs i. Often of therapeutic benefit; however, may lead to other diseases ii. Physicians are usually cautious about prescribing this therapy for a prolonged period. c. Physical or mental stress has been shown to decrease white blood cell and lymphocyte function. d. AIDS: Caused by the RNA retrovirus HIV, which binds to helper T cells, infecting and killing them 4. Treatment of immunodeficiencies includes: a. Replacement therapy available for some types i. Intravenous gamma globulin: Used in therapy of a number of immunologic disorders of the nervous system ii. Bone marrow transplantation: Used in persons with acquired causes of immunodeficiency iii. Transfusions XIII. Stress and Disease A. Stress is the medical term for a wide range of strong external stimuli that can cause a physiologic response. 1. Physiologic stress: A change that makes it necessary for the cells of the body to adapt a. Three concepts include: i. The stressor ii. Stressor’s effects on the body iii. Body’s response to the stress b. Usually response to stress is appropriate and beneficial. i. Unchecked stress response can result in deleterious outcomes, including: (a) Chemical dependency (b) Heart attack (c) Stroke (d) Depression (e) Headache (f) Abdominal pain B. General adaptation syndrome 1. Identified by Hans Selye in the 1920s 2. Characterizes a three-stage reaction to stressors a. Stage 1: Alarm i. Body reacts by releasing catecholamines, such as epinephrine (adrenaline), norepinephrine (noradrenaline), and dopamine (a) Adrenaline: Acts as a neurotransmitter in the CNS and as a hormone in the blood (b) Noradrenaline: Primarily a neurotransmitter of the peripheral sympathetic nervous system ii. Catecholamines activate the sympathetic nervous system by binding to receptor sites. (a) Alpha receptor activation results in vasoconstriction. (b) Beta receptor activation results in increased heart rate, increased force of contraction, and increased conduction velocity. iii. Other effects include increased respiratory rate, decreased blood flow to the skin, smooth muscle constriction, and various effects on the liver that increase the body’s use of glucose. iv. “Fight-or-flight” response can either prepare the body to deal with stress or weaken the immune system b. Stage 2: Resistance i. Body’s way of adapting to stressors ii. Stimulates the adrenal gland to secrete two types of corticosteroid hormones that increase blood glucose level and maintain blood pressure: (a) Glucocorticoids (b) Mineralocorticoids iii. Hypothalamus stimulates the anterior pituitary gland to release adrenocorticotropic hormone (ACTH) (a) ACTH targets the adrenal cortex to release cortisol. (1) Stimulates cells to increase energy production (2) Increases serum glucose levels and impairs the use of glucose by peripheral tissues (3) Reduces inflammation when its served its purpose (4) Increases red blood cell production and affects electrolyte levels (5) Decreases size of lymphoid tissue, which plays a role in immunity iv. Other hormones related to stress include: (a) Endorphins: Reduce pain and stress (b) Growth hormone: Promotes cell and tissue growth and repair; reduced in times of stress (c) Prolactin: Levels increase more in people with ineffective coping mechanisms (d) Testosterone: When cortisol levels are high, testosterone levels are low. v. Cortisol levels and sympathetic nervous system should return to normal during this stage. vi. Continuation of stress and accompanying corticosteroid release lead to fatigue, lapses in concentration, irritability, lethargy, depression, and a depressed immune system. c. Stage 3: Exhaustion i. Adrenal glands become depleted, diminishing the level of blood glucose ii. Results in: (a) Decreased stress tolerance (b) Progressive mental and physical exhaustion (c) Illness (d) Collapse iii. The body’s immune system is compromised, reducing the ability to resist disease. C. Effects of chronic stress 1. Hypothalamic-pituitary-adrenal axis (HPA axis): Major part of the neuroendocrine system that controls reactions to stress a Triggers a set of interactions among the glands, hormones, and parts of the midbrain that mediate the general adaptation syndrome b. Continued stress leads to loss of these normal control mechanisms i. Continued production of cortisol exhausts the stress mechanism and leads to fatigue and depression. (a) Consistently high levels lead to suppression of the immune system. 2. Stress and depression have a negative effect on the immune system. a. Causes the body to lost its ability to fight disease b. Encourages the body to release fat and cholesterol into the bloodstream, eventually causing heart attacks and stroke c. Related conditions include: i. Depression ii. Headaches iii. Insomnia iv. Ulcers v. Diuresis vi. Acne vii. Diabetes mellitus viii. Rheumatoid arthritis ix. Asthma 3. Coping mechanisms play a role in the physiologic response to stress. a. Healthy person may manage stress with very little impact on the immune system b. Patient with ineffective coping mechanisms will have deleterious effects on immune status c. Effects are worse on a person with an already compromised immune system 4. Immune suppression can be corrected with: a. Psychotherapy b. Medication c. Other positive influences that restore hope and a feeling of self-esteem XIV. Summary A. Pathophysiology is the study of the functioning of an organism in the presence of disease. B. All cells except red blood cells and platelets have three main components: a nucleus, cytoplasm, and a cell membrane. C. There are four major tissue types: epithelial tissue, connective tissue, muscle tissue, and nervous tissue. D. When cells are exposed to adverse conditions, they undergo a process of temporary or permanent adaptation to protect themselves from injury. E. The cellular environment refers to the distribution of cells, molecules, and fluids throughout the body. It is controlled by variables such as age, exercise, pregnancy, medications, nutrition and hydration, disease, and injury. F. Electrolytes in body fluids include sodium, chloride, potassium, calcium, phosphorus, and magnesium. G. pH is a measurement of the hydrogen ion concentration—that is, the acidity or alkalinity—of a solution, such as the blood. H. Cellular injury is caused by hypoxia, chemical exposure, infectious agents, inappropriate immunologic responses, inflammatory responses, genetic factors, nutritional imbalances, physical agents such as radiation, and adverse conditions such as extreme cold. I. Inflammatory response is characterized by both local and systemic effects. The outcome of an inflammation depends on how much tissue damage has resulted from the inflammation. J. Age- and sex-associated factors interact with a combination of genetic and environmental factors, life style, and anatomic or hormonal differences to cause disease. K. Analyzing disease risk involves consideration of disease rates (incidence, prevalence, morbidity, and mortality) and controllable and uncontrollable disease risk factors (causal and noncausal). L. A true genetic risk is passed through generations on a gene. In contrast, a familial tendency may cluster in family groups despite lack of evidence for heritable gene-associated abnormalities. M. In autosomal dominant inheritance, a person needs to inherit only one copy of a particular form of a gene to show the trait. In autosomal recessive inheritance, the person must inherit two copies of a particular form of a gene to show the trait. N. Immunologic diseases occur because of hyperactivity or hypoactivity of the immune system. Allergies are acquired following initial exposure to a stimulant known as an allergen. O. Perfusion is the delivery of oxygen and nutrients to cells, organs, and tissues through the circulatory system. Hypoperfusion occurs when the level of tissue perfusion falls below normal. P. Shock is an abnormal state associated with inadequate oxygen and nutrient delivery to the metabolic apparatus of the cell, resulting in an impairment of cellular metabolism. Q. Central shock consists of cardiogenic shock and obstructive shock. Cardiogenic shock occurs when the heart cannot circulate enough blood to maintain adequate peripheral oxygen delivery. Obstructive shock occurs when blood flow within the heart or great vessels (aorta and pulmonary vein) becomes blocked. R. Peripheral shock includes hypovolemic shock and distributive shock. In hypovolemic shock, the circulating blood volume is insufficient to deliver adequate oxygen and nutrients to the body. Distributive shock occurs when there is widespread dilation of the resistance vessels (small arterioles), the capacitance vessels (small venules), or both. S. Multiple organ dysfunction syndrome (MODS) occurs in acutely ill patients and is characterized by the dysfunction of two or more organs that were not affected by the physiologic insult for which the patient was initially being treated. T. The immune system includes all of the structures and processes that mount a defense against foreign substances and disease-causing agents. U. The body has three lines of defense: anatomic barriers, the inflammatory response, and the immune response. V. The two anatomic components of the immune system are the lymphoid tissues and the cells responsible for mounting an immune response. W. The primary cells of the immune system are the white blood cells, or leukocytes. X. There are two general types of immune response: native and acquired. Y. Immunity may be humoral or cell-mediated. Z. Important white blood cells in the immune system include neutrophils, eosinophils, basophils, monocytes, and lymphocytes. Other important cells of the immune system include macrophages, mast cells, plasma cells, B cells, and T cells. AA. The antibodies secreted by B cells are called immunoglobulins. Antibodies make antigens more visible to the immune system in three ways: by acting as opsonins, by making antigens clump, and by inactivating bacterial toxins. BB. The inflammatory response is the reaction of the body’s tissues to cellular injury. It is characterized by pain, swelling, redness, and heat. CC. The two most common causes of inflammation are infection and injury. DD. The plasma protein systems that modulate the inflammatory process include the complement system, the coagulation (clotting) system, and the kinin system. EE. Cytokines are products of cells that affect the functioning of other cells; they include interleukins, lymphokines, and interferon. FF. Chronic inflammatory responses are usually caused by an unsuccessful acute inflammatory response after the invasion of a foreign body, a persistent infection, or an antigen. GG. Normal wound healing involves four steps: repair of damaged tissue, removal of inflammatory debris, restoration of tissues to a normal state, and regeneration of cells. HH. Wounds may heal by primary or secondary intention. Healing by primary intention occurs in clean wounds with opposed margins. Wounds that heal by secondary intention have a prolonged inflammatory phase and more abundant granulation tissue. II. Hypersensitivity is an increased response of the body to any substance to which the person is abnormally sensitive. A hypersensitivity reaction may be immediate or delayed. JJ. Hypersensitivity reactions may be classified as autoimmune, idiopathic, or blood incompatibility reactions. KK. Immunodeficiency may be congenital or acquired. LL. Stress does not cause death directly, but it can permit diseases to flourish, ultimately leading to death. MM. The general adaptation syndrome describes the body’s short-term and long-term reactions to stress. NN. Stress causes the sympathetic nervous system to be stimulated. This occurs through release of catecholamines that activate the sympathetic nervous system by binding to alpha and beta receptor sites, resulting in effects categorized as the fight-or-flight response. OO. Stress also causes secretion of cortisol, which has many useful effects. However, continuous secretion of cortisol has deleterious effects. Post-Lecture This section contains various student-centered end-of-chapter activities designed as enhancements to the instructor’s presentation. As time permits, these activities may be presented in class. They are also designed to be used as homework activities. Assessment in Action This activity is designed to assist the student in gaining a further understanding of issues surrounding the provision of prehospital care. The activity incorporates both critical thinking and application of paramedic knowledge. Instructor Directions 1. Direct students to read the “Assessment in Action” scenario located in the Prep Kit at the end of Chapter 8. 2. Direct students to read and individually answer the quiz questions at the end of the scenario. Allow approximately 10 minutes for this part of the activity. Facilitate a class review and dialogue of the answers, allowing students to correct responses as may be needed. Use the quiz question answers noted below to assist in building this review. Allow approximately 10 minutes for this part of the activity. 3. You may wish to ask students to complete the activity on their own and turn in their answers on a separate piece of paper. Answers to Assessment in Action Questions 1. Answer: B. stroke volume Rationale: Pulses are a pressure wave that travels through the arterial system with the contraction of the left ventricle. There must be a sufficient amount of blood traveling through the arterial system, and there must be sufficient force propelling the fluid through the system. The pulse is primarily the result of stroke volume, which is the amount of blood ejected from a ventricle in one contraction. 2. Answer: B. wheezing and widespread vasodilation Rationale: Anaphylactic shock is the result of an extreme allergic reaction in which the immune system responds to neutralize the invader. These are systemic reactions that affect more than one body system. The cardiovascular system reacts by dilating the blood vessels and dropping the patient’s blood pressure, causing hypoperfusion. The respiratory system contracts the airways, making them smaller, which accounts for the whistling sound known as wheezing. In later stages of anaphylaxis, there will be little air movement at all due to the constriction. 3. Answer: A. anaerobic metabolism Rationale: In medical terminology, the prefix “an” means “without.” Anaerobic means “without air.” Cells within the body can function for a short period in an anaerobic state. Hypoxia is the main cause of anaerobic metabolism. Cells that operate in this manner will produce an excess of lactic acid, which will eventually destroy the cell if the hypoxic state is not corrected. 4. Answer: B. blood pools in expanded vascular structures Rationale: As the name implies, distributive shock occurs when there is a problem with transporting blood through the body’s distribution system because the distribution system expanded without a corresponding increase in fluid. For example, pouring 1 cup of water into a vessel designed for 1 cup will fill it. Pouring 1 cup of fluid into a 3-cup container will leave the vessel two thirds empty. The body cannot manufacture more blood on a large scale to make a difference in circulating volume, so perfusion of the tissues drops. 5. Answer: B. Increased systemic vascular resistance Rationale: When the body senses that there is a problem due to lack of nutrients reaching the cells, it reacts by trying to correct the problem. This happens through the release of catecholamines and by increasing vascular resistance. Essentially the body is trying to move the fluid faster and decrease the size of the container. 6. Answer: A. diminished lung sounds Rationale: Anaphylactic patients, who have little to no air movement, are in extremis. There is little air moving, which is not effective to sustain life for long. Providing aggressive airway management as quickly as possible before this state occurs will offer the patient the best chance for survival. Additional Questions 7. Rationale: The patient with a high PaCO2 and a low pH is in respiratory acidosis. Arterial blood gas (ABG) readings are necessary to accurately determine the patient’s condition. ABG readings were typically not available in the field setting until recent advancements in technology. Currently, field-rated blood sampling devices are available for purchase. Without field ABG readings, paramedics can still make a reasonable assumption if a patient is severely hypoxic. 8. Rationale: A patient who eliminates an excess of carbon dioxide will develop signs and symptoms of respiratory alkalosis. Arterial blood gas levels will show a low PaCO2 and a normal or slightly high PaO2. Patients will show signs of numbness, chest tightness, tetany, and dizziness. Assignments A. Review all materials from this lesson and be prepared for a lesson quiz to be administered (date to be determined by instructor). B. Read Chapter 9, Life Span Development, for the next class session. Unit Assessment Keyed for Instructors 1. What are the three main components of cells found in higher organisms? Answer: Nearly all cells of higher organisms, except mature red blood cells and platelets, have three main components: Cell membrane; cytoplasm, which contains the cell’s internal components, or organelles; and a nucleus. The cell membrane consists of fat and protein. It surrounds the cell and protects the nucleus and the organelles, functional structures within the cell’s cytoplasm (fluid). The organelles operate in a cooperative and organized manner to maintain the life of the cell. (p 335) 2. What are the three types of muscle structure, and what are their individual functions? Answer: These structural and functional classifications are combined in the categorization of muscle into three types: Skeletal muscle (striated voluntary): Most of the muscles used voluntarily in day-to-day activities are skeletal muscles. Cardiac muscle (striated involuntary): The heart consists of cardiac muscle and has contractile ability and the ability to generate electrical impulses. Smooth muscle (nonstriated involuntary): Smooth muscle lines most glands, digestive organs, lower airways, and vessels. When a patient’s brain senses the need to respond to an environmental stimulus by vasoconstriction, the vessels in the periphery react. For example, the smooth muscle of the bronchioles may constrict during an asthma attack, causing the person to wheeze and making it difficult for him or her to expel air from the lungs. Smooth muscle in the iris is responsible for constriction and dilation of the pupil in response to changing light conditions. (p 337) 3. Which nerve receptors are stimulated by epinephrine and norepinephrine? Answer: Adrenergic receptors are associated with the sympathetic nerves and are stimulated by epinephrine and norepinephrine. Activation of an adrenergic receptor causes a sympathetic response, such as vasoconstriction or vasodilation. Alpha receptors and beta receptors are in the adrenergic class. (p 337) 4. Describe the mechanism whereby endocrine and exocrine hormones reach their specific target organ. Answer: Endocrine hormones (such as thyroid hormones and adrenal steroids) are carried to their target organ or cell group in the blood. Exocrine hormones reach their target via a specific duct that opens into an organ; examples of exocrine secretions include stomach acids and perspiration. (p 340) 5. Describe the movement of a solvent through osmotic pressure. Answer: This is the movement of a solvent, such as water, from an area of low solute concentration to one of high concentration through a selectively permeable membrane to equalize the solute concentration on both sides of the membrane. Osmotic pressure develops when two solutions of different concentrations are separated by a semipermeable membrane. Water moves from the region of low osmotic pressure to the region of higher osmotic pressure. (p 341) 6. How does edema occur? Answer: Edema occurs when excess fluid builds up in the interstitial space. Peripheral edema (as in the ankles and feet) is the most common form. Severe edema may be caused by long-standing lymphatic obstruction. If a person is unable to get out of bed for an extended period, edema may occur in the sacral area (sacral edema). Ascites is the abnormal accumulation of fluid in the peritoneal cavity. (p 342) 7. What is hypokalemia? What signs and symptoms of hypokalemia would you expect to find? Answer: Hypokalemia is defined as a decreased serum potassium level. Common causes include decreased potassium intake, potassium shifts into the cells (related to insulin, alkalosis, or beta-adrenergic stimulation such as with epinephrine), renal potassium losses (such as with increased aldosterone activity and diuretic use), and extrarenal potassium losses (such as with vomiting, diarrhea, and laxative use). Muscular weakness, fatigue, and muscle cramps are the most frequent complaints in mild to moderate hypokalemia. Flaccid paralysis, hyporeflexia, and tetany may occur with a very low level of potassium (< 2.5 mEq/L). The ECG shows decreased amplitude and broadening of T waves, prominent U waves, premature ventricular contractions, and other dysrhythmias (such as torsades de pointes), and depressed ST segments. (p 344) 8. What does this equation represent? H2CO3 ? CO2 + H2O? Answer: The fastest way the body can eliminate excess H+ ions is to create water and carbon dioxide, which can be expelled as gases from the lungs. The equation seen above illustrates this process, which occurs in the lungs. Acidosis can develop as a result of abnormal respiratory function, including bradypnea, tachypnea, labored breathing, or shallow breathing (reduced tidal volume). The following equation demonstrates this: ? Respirations ? ? CO2 ? ? H2CO3 ? Acidosis. At the other end of the acid-base spectrum, alkalosis can develop if the respiratory rate is too high or the volume too large, as shown in the following equation: ? Respirations ? ? CO2 ? ? H2CO3 ? Alkalosis (p 349) 9. What is virulence? Answer: Virulence measures the disease-causing ability of a microorganism. The pathogenicity of any particular microorganism is a function of its ability to reproduce and cause disease within the human body. In particular, the growth and survival of bacteria in the body depend on the effectiveness of the body’s own defense mechanisms and on the bacteria’s ability to resist the mechanisms. A depressed immune system is less capable of fighting off microorganisms that the body perceives as harmful; populations with weaker immune systems include newborn infants, older adults, people with diabetes, and people with cancer or other chronic diseases (p 354) 10. What is MODS? Answer: Multiple organ dysfunction syndrome (MODS), first described in 1975, is a progressive condition that occurs in some critically ill patients. It is characterized by the concurrent failure of two or more organs or organ systems that were initially unharmed by the acute disorder or injury that caused the patient’s current illness. Six organ systems are surveyed in diagnosing MODS: respiratory, hepatic, renal, hematologic, neurologic, and cardiovascular. (p 370) Unit Assessment 1. What are the three main components of cells found in higher organisms? 2. What are the three types of muscle structure, and what are their individual functions? 3. Which nerve receptors are stimulated by epinephrine and norepinephrine? 4. Describe the mechanism whereby endocrine and exocrine hormones reach their specific target organ. 5. Describe the movement of a solvent through osmotic pressure. 6. How does edema occur? 7. What is hypokalemia? What signs and symptoms of hypokalemia would you expect to find? 8. What does this equasion represent? H2CO3 ? CO2 + H2O? 9. What is virulence? 10. What is MODS?

Related Downloads
Explore
Post your homework questions and get free online help from our incredible volunteers
  1040 People Browsing
Your Opinion
Where do you get your textbooks?
Votes: 372

Previous poll results: How often do you eat-out per week?